You are on page 1of 152

ALL AVAILABLE SEPTEMBER PAPERS ON GROUP

IN ONE FILE BY PORTA HEPATIS

RADIOLOGY

5 SEPT EVENING SHIFT

Paper 2 questions..

This file contains:RADIOLOGY 5 SEPT EVENING SHIFT


Paper 2 questions..
7 SEPTEMBER MORNING SHIFT..
MED AND ALLIED..DR SAHAR RAZA
Ophthalmology FCPS Part One (5th Sep,2016) Paper 1+2
RADIOLOGY 1st SHIFT 5 SEP 2016
Gynae and Obs 7th September 2016 (evening)
Surgery paper 2016 5th Sep 2016 EVENING

1.What cause hypercalcemia n renal failure ?


A. Parathyroid adenoma
B. Hypervitamin D
2. Metastatic calcification?
A. Normal kidney
B. Follicular adenoma thyroid
C. Malarial parasite
D. Tb
3. Patient with sciatica. Compression on 5th lumbar segment causes
A. Absent ankle jerk
B. Brisk ankle
C. Absent babinski
D. Weak dorsi flexion
E . Weak planter flexion
4. About intercostal nerves
A. 12 in numbr
B. Supply skin of whole body wall
C. Suply abdominal muscles only
D. Part of symphathetic chain
5. About esophagus
A. Suplied by thoracic aorta only
B. Lies on right side thruout its course
C. Cardiac orifice at 10th costal cartilage
D. Serosa at lower end is thick somethng like that
6. About spleen
A. Develops in ventral mesogastrium..i did this..dont remember the rest of options..they were really
confusing
7. Another questn about spleen
A. On CT visveral surface on medial side..somethng like this..dont remember reat of options..they
were confusing
8. About right brachiocephalic vein
A. Begins behind right sternoclavicular joint
B. No valves
C. Shorter than left
D. Vertical
E. Ends at i dont what level was mentioned..yes First 4 options were correct..only one option was

wrong..shaid Stem me EXCEPT likhna bhool gae thay.


9. Optic canal
A. Lesser wing of sphenoid
B. Greater wing of shpenoid
10. Descending thoracic aorta passes behind
A. Median arcuate ligament
B. Medial arcuate ligament
C. Lateral arcuate lig
11. One questn about femoral hernia..like why it strangulates..
A. Excessive movemnet lower limb
B. Pressure by inguinal lig
C. Less space in femarl canal
D. Tight boundaries of femoral ring
12. Wo femous question pelvis sink wala...pelvis sink to left side..cause
A..right side gluteus maximus n medius
B. Right side gluteus max, medius, minimus
Rest of options were of left side muscles
13.. multifactorial inheritance?
A. Sickel cell anemia
B. Cystic fibrosis
C .omphaloclee
D. Alkaptonuria
14..resistant to stomach acid?
A. Vibrio choleera..dont remembr rest of the options
15. Radial artry is accompanied by?
A.median nerve
B. Musculocutaneius nerve
C . Deep brach of radial nerve
D. Sup brach of radial nerve
16..ek long scenario..L2 level mention tha on right side..n histological feature mention tha k 2 black
line..pta nai aesa e kuch tha ..confusing ..options were
A. Colon
B. Stomach
C. Ileum
D. Jejunum
E. Rectum
17. Motor loss on right side of body..but forehead spared..level of lesion
A. Anterior limb of internal capsule
B. Genu

C. Genu + posterior part poterior limb of internal capsule


D . Anterior part of posterior limb internal capsule
E. Genu + anterior part of postr limb of internal capsule
18. Sensory loss on left side face n left side body. Level of lesion
A. Thalamus
B. Pons
C. Internal capsule
D. Medulla
19..ek aur scenario about facial nerve lesion..asked about level of lesion
A. Apex of petrous part temporal bone
B. Anterior part petrous bone
C. Posterior part petrous bone
D. Superior part of petrous temporal bone
20. Inferior epigastric artry lies lateral to
A. Direct inguinal hernia
B. Indirect inguinal hernia
21..lymphatic of gallbladder first drain into
A. Preaortic lymphd node
B. Para aortic
C. Porta hepatis lymph node
22. Lymphatic from visceral surface liver drain into
A. Poaterior mediastinal lymph node
B. Pre aortic lymph node
C. Para aortic
23. Ek long scenario about lesion in perineal area tha koi..lymphatic spread pochi hui thi k kidar se aa
sakti..
A. Cervix
B. Vagina above hymen
C. Body of uterus
D. Uteurs above uterine tube
e. Ovaries
24. About slowly adapting sensory receptors..
..all options were about action potential..kisi me rapid kisi me slow..kisi me no effect on action
potential...confusing one

25. Ek long scenario..ended up asking parasympathetic supply of descending colon n sigmoid


A. L5 l4

b. S1 s2 s3
c. S2 s3 s4
d. S3 s4 s5
26.in one questn they asked LCA supply..i did bundle of his..none of the other options were
corrct..this was the close one

27.. Anterior mediastinum


A. Thymus
B trachea
C. Vagus
D. Lhrenic
E esohgus
28.. Surgery..submandibular region..muscle paralysed
A. Leteral pterygoid
B. Medial pterygoid
C. Massetr
D. Buccinatr
E .temporalis
29..counterpart of rete testis
A. Paramesonehpric duct
B. Mesonephric duxt
C. Mesonephric tubule
30..Most imp cell of chronic inflamation.
A. Lymphocytes
B. Macrophages
Yes they both were present in dif options.
31. max number of sodium channel present
A. Initial segment of axon .i did this
B. Node of renvier
32. Not a part of femoral triangle
A. Obturator nerve
B femiral nerve
C. Femoral sheath
D. Addutr longus
33.. At what level glucose begin to appear ib urine
A. 180

B 200
C. 250
34. What lies posterior to periaqueductal grey area
A. Substantia nigra
B. Red nucles
C. Pineal gland
D. Colliculus
35.Fatty diet affect which organ most
A. Kidney
B. Brain
C. Liver
D. Skin
36..all time fav cpsp controversial questns were there..cause of fatty liver in pak
A. Fatty diet
B. Protein malnutritin
C. Hep b n c
D.Alcohol
37.. Breast atrophy in adult
A. Estrogn
B. Progestrone
C. Both..i did both
38.. ek confusing questn tha..about what lies close to pericardium..aesa e kuch tha
A. Sa node
B. Av node
C. Bundle of his
D. Purkinje
39 .ek IVC k relations pe tha..
A. Lies close to 3rd part duodenum
B. Cross by ileum mesentry
40. lymph from nipple breast
A. Anterior axilary
B. Posterior axillary
41. Superficial dorsal vein of penis drain into
A. Great sapheouns vein
B. Internal iliac vein
42. Which segment diluted urine
A. Thick ascending limb
B. Dct

43. Mi ..post iv septum..which artry.


A. Lca
B. Lad
C. Rca
44. Question about hemophilia.
a. X chromosome
b. Y chromosome
C. Chromosome 6
45. pancreatic fluid accumulates in the
A. Paracolic gutter
B.subprhenic
C. lesser sac
D. subdiaphramg
47.. uncinate process of pancreas..what pases anteriorly
a. Superior mesenteric artry
B. Inferior mesenteric
C aorta
d. Celiac artry

48. Aortic arch aneurysm cause compression of


A. Esophgus
B. Left bronchus
49..patient unable to turn foot medially but can turn laterally..muscle paralsd..
a. Tibialis anterior n extensor hallusis longus
b tibialis anterior and tibialis posterior
c . Tibialis posterior n flexor hallucis longus

50. Length of thoracic duct


a. 38cm
b.40cm
C. 45cm

51. No of segments of kidney


a. 5
B. 6
c. 7
D. 8

52.. sequence of renal artry


a. Lobar artery ..arcuate artry ..interlobular artery
b. Segmental artery.. lobar artery..inter lobar artry ..arcuate..interlobular
c. Lobar artery..interlobular artery .arcuate artry..interlobar artry

53. Sciatic nerve pases thru


A. Lesser sciatic foramen
B. Greater sciatic foramen
C. Obturator foramen
D. Behind inguinal lig

54..arterial supply of jejunm


A. Celiac artry
B.splenic artery
c. Sup mesenteric artry
d. Inferic esenteric artry

55. Lymph from testis drain into


A.. para aortic lymphnode
B. Pre aortic lymph
C. Superficial inguinal lymph node
d. Deep inguinal lymph node

56. Patient with..dependant edema..dilation of abdominal veins..hepatoma cause compression of


A. IVC

b. Hepatic veins
C. Portal vein
d. Splenic vein

57. Question about bile duct.


A. 6 inch long
b. Present on right side of hepatic artery in hepatogastric ligament
C.

All these questions are from ppr 2..Ppr 1 was repeated !!


Regards!

7 SEPTEMBER MORNING SHIFT..


MED AND ALLIED..DR SAHAR RAZA..

(i have compiled which I remember feel free if any


mistakes. Human errors occur)
1.theophyline interaction with which causes dec its clearance.
a.adenosine b.cimitidine c.beta bloker I think
2.most common premalignnt condiotion.
a.leukoplakia b.erythroplakia c.lichen planus d.apthos ulcer
plzz concentrate on these stems about change ,,lesion and
condition
3.painful vasicular eruption at chin and dermatomes involve
a drug given like tgo4 pata nh is tarh k but affect little..
diagnosis
1.trigeminal neuralgia 2.herpes
4. immunoglobulins produced by plasma cells
5. following produce by plasma cells
1.albumin 2.firinogen 3.gama globulin
6. an enzyme which combine two and make a single
compounda. synthetase b. isomerase c.transferase
7.Intercoastal nerves
a.only supply skin

b.12 in number
c.Supply only intercostal muscles
d.Associated with sympathetic chain
I do c bcz not sure about b option
8.percentage of lichen planus chance to covert in malignancy
a.1-10%
b10-15
c.15-20
d.20-25
e.25-30
people say it 15 20 % but I searched alott on net on every site
everywhere I found 1 2% thatswhy I prefer a optionconfirm
with senior.
9 . after tooth extraction cause of sub acute endo carditis
a.vrirdans .
b staph aurus
c.fecalis
10 another of subacute endocartis repeat ans viridins
11. Blood born infection
A. staph
b.hep b
other was irrrlevant
12. Child wd sore throat and fever u gve antibiotics...after 3
weeks hematuria occure....smethng like that scenario.
1.good pasture syd
2.poststreptococal glomerulonephritis

13.Increase LES tone and speeds up gastric


emptying......metaclopromide
14.thee is achalasia of lower esophagus
a. no myentric plexus in lower esophagus
b.food rapidly goes through esophagus something like that..
15. Dignosis for Hashimotos
a.antimitochondraial antibody..
b. Antimicrosomal and anti thyroglobulin
16. thyrocervical trunk..
a. suprascapular branch
b. arise from second part of subclavian
c.superior intercostal is branch..
17. High carbohydrate diet taking should have in diet or
deficint in
a.thiamine
b.riboflavin
18. Muscle of back supplied?
a. dorsal rami of cervical spinal.
19. Cavernous sinus contain
A. abducent nerve
b.trochlear
c.oculomotor
20.premalingnat nevus is
a.compound nevus

b.intradermal nevus.
There is no option of dysplastic or congenital giant nevus..
21.hyperthyroid ,goiter and exophthalmos
a.toxic goiter
b. graves disease
22. Alchohol cause def of which vitamin.
A, vit b
b.vit a
c.vit d
23. Sle k diagnostic test
a.anti double strand antibody
24. Sjogren ass wd whith which?
a.sle
b. RA
25.lp taken at which site
a. above l4
b. below l5
26. baby has abdominal pain and anemia
diagnosis.....repeated scnerio of sickle
a.hbss
b.hb a
c.hb cs
27. Hyperbaric oxygen use in treatment of sea divers..
a.barotrauma

b.decompresion sickness
28. optic chiasma dmage with also hypothalmus damge to
kia hoga sath ma ...
a.dibetes inspidus..
b. hyperphgia
c.hypertnsion.
29. Man has nan lethal MI wat happens to him in few
minutes?
A.compete recovery without any complication
b.vetricular arrhythmia
30What decreases penicillin excretion from kidney
a.Probenecid
b.Erythromycin
c.Gentamicin
d.Benzathin penicillin
e.Penicillin G
31.Dead space decrease in
A.standing
b.tracheostomy
c. deep insp
d.shalow breathing

32. SA node action potential due to


a.Na influx
b.Ca influx
33. Large intestine most accurate
a.Caecum may hv a dia of 9cm
b.Transverse colon supplied by right colic
c.Appendix is rarely retro caecal
d.20% hv mesentry attached to ascending colon
e.Descending colon lies medial to left kidney
Cecum I think 3cm ha, transverse colon ko midle colic karte ha
pr appendix most common site retrocecal he ha. Desending and
lies also at lateral not medial thatswhy I prefer d option bcz I
study that 15 % of ascending colon in some adults have
mesentry kindy discus with ur seniors..
34. MOA of methyldopa
Binds adrenergic receptors in cns
Acts directly on arterioles
35. Left axis deviation in
a.Right sided myocardial damage
b.RBBB
36. taking blood sample at cuboital fossa wch nrve injured?
a. median
b.ulner

c.radial
37. Common peroneal nerve lesion
a. Inable to evert
b .Inable to invert
c. Inable to planter flex ankle n big toe
d Loss of sensation on medial side of leg
38. A child has bald patch on scalp since birth. Which is
leathery. On microscopic exam there is papillomatous
projections in epidermis. Dermis main abundance of
sebaceous glands.. Cells k baray main b kch description
the What is the serious complication
a.Bcc
b.Sq. Cc
c.Keratocanthoma
d.scaling alopecia
39. Mgso4 action inhibited by
a.Na
b.Cacl2
c.K
40. Thiazides decreses clearence of
a.Class 2 antidepressants
b.Phenothiazide antipsychotics
c.TCA
d. lithum
41. 1 scenario that pt cant flex index finger and loss
sensation os lat palmer surface.wch nrve damage
a.median

b.radial
c.ulner
42. Heparin primary action achieved by binding to
a.Antithrombin 3
b.Xa
c.Thrombin
43. Skin incision with sharp object redness due to
a.Axonal reflex
b.Histamine
44. Angina may be worsened by
A. Atropine
B. Salbutamol
C. Theophyline
D. Vassopressin
E. Verapamil
I mark c
45. Most imp feature of shock
a.tissue hypoxia
b.hypotension
I mark a
46.bar body in xxx
a.1
b.2
c.3

47. Short arms /legs . Lordosis . With normal mental health


. Disease mode of transmission.. (achondroplasia)
a.aut dominat
b.aut recesive
c. xlinked
48. adult Polycystic kidney disease mode of transmission
a. aut dominant
b. aut recesive
49. L depo given in Parkinsonism
a. dec drug inactivation in periphery
50. Vomiting present and absent . ..repeated
a.Ratio
b.Nominal
c.Ordinal
d.Pametrics
51.hormone increase in response of meal in duodenum
a.ckk
52.increase bladder contractlity.
a.cck
53. Heparin acts by binding to
a.Factor xa n thrombin
b.Factor xa only
c.. V|||
d.Factor v||

54. In pancreatic fistula action of somatostatin???


1. Inhibit enzymes
2. Inhibit secretion,,,,
i mark b
55. Inversion and eversion of foot on which joint
a.subtarlar joint

56. Phrenic nerve relation to ant to which muscle


a.scalnues ant
b.scalneus medias
57. Phrenic nerve root value
a.C3c4c5
58. Pregnant lady go into obstructed labour and reach
hospt 50 miles.when reachd hosptl she died....cause?
A.Fat emboli
b.amniotic emboli

Aspirin should be stopped before surgery. How many days the effect
ofaspirin remains?
59.

A. 7-9 days
B. 3-4 days
C. 10-12 days
D. 14-16 days
I did a..seen in group majorty ans is a

60. Heart chamber related to sternum(costochondral surface)


a.rt ventricle
b.rt atrim
c.left ventricle
61. Regarding blood suply of bones
a.Small bones suplied yad nh.
b.Irregular bone ka bh tha
c.Flat bones bone suplied by nutrient
d. Shaft of long bone suplied by Nutrient artery only.
e.ends of long bones suplied by epiphyseal artries.
I did e.. bcz see in past papers
62.urinary bladder mass there and features towards
squamous cell ca.cause
a.smoking
no option of shistosomias
63.one of cv raised 107 and gut problem..
Megaloblastic anemia
64. severe steatorrhea
a.pancreatic resection
65.nutmeg liver..
a.chronic passive congestion
66.3rd heart sound murmr occur due to
a.rapid inflow in ventrcles

67. one was esophgeal ca and uper 3 ma to squomus


diagnose and ,,,, option ma smoking nh tha ...
a.alcholoism
other cannt recall no option smoking thatswhy choose
a
68.eeg about change tha
a.alpha waves disapper on eyes open or kuch better nh
laga cannt recall
69.hyoid bone level.
C3
70.humman intermediate host of
A.Hydatid disease
71. pt died on brain which type of necrosis
a.liqufctive necrosis
b.coagulative necrosis
72.about psudomans
a.endtoxin
73.aplastic anemia
a.hypocelluar tap on trephine biopsy
74.prophylaxis agaist tetnus to population
a.toxoid

b. antitoxin
75.thamalic stoke or hamrohge like scnerio
a. hperasthesia I mark
b.Synesthesia like option
76.red infarct..
a.organ have dual blood supply
77.yellow fever in non endemic area
a. epidemic
b. sporadic
c.endemic
78.ventricles completely depolarized at
a.qrs
b. st
79. pt cant speak or utter words damage to
a. wernickes
b.brocas
80.. hemartoma
A. mass of mature.but disorganised tissue.indiginous
81.left precentral gyri damage,,,,
a.rt hemiplegia
82.adenoma..
a.Glandular pattern

83.phledelphia chromosomes
A.cml
84. Atrial flutter (repeated bcq)
a.Treated with defibrillation shock therapy
b. rate is so rapid above 400beats/min
c.Heart beat between 200 and 350beats /min
85.creatinine clearance
Values u .creat.. 144 v.. 1.4
And plasma creat=1.2
a.1.68
b.168
c.163
d.cannot be calculated
86.true hermaphrodite, the most cpsp fav
a.xy
b.xyy
c.xxy
87.xerostomia ,, keratitis
a.sjorjen syndrome.
88 .. one was k pt k eye ma mydriasis and ptosis ha..
whtas cause
a.occulomotor damage

b.horner synd
I do a.. bcz horner ma miosi hote ha inocculo damage
mydriasis with ptosis occur
89.In G6PD what is present?
a.comb test ,
b. to osmotic fragility
c.,heins body
d.microcytosis,or long cells
90.In venous blood rbc hav?
a.Inc cl conc
b.Inc hco3
91. In deep sleep
a. serotonin
b.melatonin
92.diabetic patient ..repeat
a.neuropathy and angiopathy..
93.mg store in
a.bone
94. Diabetic patient after gram negative infection present
with shock after 6 days sugar level normal.option yad nae
a. acute tubular necrosis
b. increase permeability
95. Pt stands frm supine pos.wat wil happen?

a. inc heart rate


b. decvenous return
96.boat shaped organism in case of pneumonia
a.pneumocystitis jervoci
97.Patient smoker psycotic with vomitin abdominal pain n
visible perstalsis in upper portion of stomach
a.Pyloric stenosis
b.Psychogenc.
c.achlasia
98.fisherman with gums hypertrophy and bleeding def of
which
a.vit c
b.vit k
99. Bilateral ptosis at the end of day is conformatory test?
A. Tensilon
B.antibodies
c.emg
100.isthamus of thyroid
a.2,3,4 rings
101.corticospinal tract decuusate at
a.pyramid
102.pus contain

a.dead bacteria
b.dead neutrophil
103.which cause malignancy in git
a.h pylori
104.which condition predispose to cancer
a. bartes esophagitis
105.hypophysis cerebri related with
a.sphenoid sinus
b.ethmoid sinus
106.soft palate lined by
a. stratified squamous non cornified
b. stratified squamous cornified.
107.pt died after 6 days of MI on autopsy large amount of fluid
seen in pericadia cavity.
a.cardiac tamponade
108.gfr increase by
a. afferent arteriolar dilation
109.post 1/3 interventrcular sptum supplied by
a. marginal

b. rca
c. circumflex
110.thirst is stimalted by.
a.increase osmolarty and dec volume
b. dec osmolarty and increae volume
111.prolactin regultation by
a.dopamine
112.about pulse preasure depends.
a.venous compliance
113.heparin ,histamine and protease like kuch tha release
from
a.mast cells
114.glucocorticoid causes dec which cell..
a. lymphocyte
b. neutrophil
115.gluconeogenesis is caused by
a.insulin
b. glucagon
116.resection of ilum

a. increase water content in feces


117.ventral corticospinal end at
a.mid thoracic
b.upper thoracic
c.all spinal cord
118.muscle stretch then relaxation by
a. muscle spindle
b.gto
I do muscle sindle bcz tension is not mention here.
119. muscle stretch to extreme which prevent from damage or
tear like that
a. muscle spindle
b.GTO
120. Hyaline cartilage
A not present in nasal septum
B pinna
C TMJ articulation
D epiglottis
e. two menisci
121.antimetabolite use in cancer treatment
a. methotrexate

b.azathioprine
b.flourouracil

122.left shift of oxyhb curve


a. incease temp
b.co poisononing
123.about left renal
a. ant to both aorta and left renal artery
124.one of delayed wound healing
a.vit c deficiency
125.intermediate filament for cancer is
a.cytokeartin
b.vimentin
126.diffusion depends upon like stem
a.total surface area
I do other cant recall
127.in myasthenia gravis treatment ya affect by asked.

a.Choline acetyl transferase


b.acetyl coA
c,acetylcholine
d.acetylcholinesterase
e.cholinestrase.
there was no option with anti cholinesterase ,,, I checked a lot of
tims the options.
128.definitive diagnosis of a pt with tb asked
a. caseous necrosis
b. identification of afb
129. length of rt bronchus
a.2.5 cm
130.in mountain the po2 and nitrogen from 700 wa asked..
a.po2 144 and pno2 546
b.po2 110 and pno2 590
c.po2 100 pno2 600
d.po2 98 and Pno2 602.
Values of o2 I am sure but not for nitogen not sure ..disscus it
with seniors

131.in copd pt some valus of abgs was asked,,,,,


a.Vt 350.fev .20000. fvc 36000
I only recall this which ido baqi sab ma vt k values zyada the
400 and 450 like that.
132.univeral recipient
a.AB NEG
B.AB POS
C.O POS
D. O NEG
133.about cerebellum
a. lies below tentorum cerebelli
b. middle cerebellar connect with medulla or spinal was
given.
c.only supply by basilary
I do a....
134.glucagon secretion increase by.
a.excerscise
b.insulin
I do A.

135.. Pt have malar rash and anti ds DNA positive now having
protinraia and hematuria
a. lupus nephritis
136 . edema of kidney origion
a. ALbuminuria and na retention
137 . a pt has injury red flare was seen due to .. repeated bcq
a.Histamine
138.one was decrease na secretion due to .
a. maximum na reasbsorption
I do othr cannt recall
139.one was asked about N REM
A.night mares
b. on eeg delata waves like option
mjhe delta waves option sahe laga other are not matchesd I
cannt recall
140. hormone of stress
a. cortisol
b. norepinephrine
I marked A. BCZ SO popular on gp..

141.gluocorticoid.
a.dec utilization of glucose

142. thyroid increases which


a. free fatty acid
b . cholesterol
I mark b ma bhol gae the hahhaha
143.micturation reflex..
a.once start then self regertive
b. control by sacral segments of spinal cord.
In ppq it was given self generative but there option change
thatswhy I choose b ,,, mjhe wo best laga
144.post interventricuar septum ( mjine recall k post 1/3 h tha
dono paper ka .. but some says1/3 not given in 2 paper)
a. rca
b. circumflex
c.lad
145.essential fatty acid
a.linolenic acid

146. . Coarctation of aorta is associated with (repeat bcq)


A. Constriction just after the subclavian artery
B. Dilatation below the coarctation
147. The most common changes which occur in a case
of uterine prolapse (repat)
A)chronic cervicitis with squamous metaplasia
B)dysplasia of cervix
C)hypertrophy of cervix
D)squamous metaplasia , endocervix
E)vaginal discharge
148. a 2 year boy having mcv raised .. and choice of
investigation . repeated bcq from past
a. vit b12
b.intrinsic factor
149.heparin given affect on
a.factor X
B.fctor x and thrombin
c.factor vii
150 .. chritsmis disease
a. Factor 9 deficient
151. by the nd of 5th week
a. heart definite form
b.stomach rotates
c.limbs bid appear..
pta nh tanveer ko kisne bata dia ha k definte heart 5 week
ma ha.or upper se logo k refrnce k tanveer dekho its

not authentic plzzz dont follow blindly heart defib=nte is


b/w 7 -8 weeks
152. Internal carotid artey
a.Latera to external carotid at origion then goes medialy .. I
marked
b.Give ophthalmic artey that enter tha eye through superior
orbital fissure
153.renin dec by.
a.adh
b. ag 2
I marked b
154.fascial nerve ( repat )
a. supplies stapedius
155. one was esophgeal ca and uper 3 ma to squomus
diagnose and ,,,, option ma smoking nh tha ...
i do alcholoism
other options cannt recall

Thanks to awais khan shumaila kiran ,,,Fatima zehra shabana


abbas,,,..sidra khan .. zain safdar rabia rafiiq and salvia fissure or
jn sabne bh recall kye ,,,,,,

OphthalmologyFCPSPartOne(5thSep,2016)Paper1+2

MaximumMcqsicouldrecallfrommyexamofophthalmologyincludingpaper1and2.Itried
mybesttowriteallthecorrectanswerswithreferencesstilltherearesomeconfusingquestions
forwhichihavementionedtheoptionsimarkedandpassed.

1) Shortestactivemusclebelly
inferioroblique(correct,ReferanceAmericanAcademyofOphthalmology)
inferiorrectus
superiorrectus
superioroblique

2) Aboutsclera
Piercedbyshortciliarynervesatlaminacribrosa
Bluishtingeinchildren(correct)
veryvascular

3) Opticnervepassesthruwhichbone
Sphenoid(correct)
Frontal
Ethmoid

4) RegardingGap43protein(veryweirdquestion..sorryforgotoptions..kindlystudyabout
itfromgoogle)

5) Regardingembryologywhichiscorrect
Conesdevelopbeforerods(imarked)
Ganglioncellscoverfoveaatbirth
Conesfullydevelopedat5months
(KindlyconfirmitfromAmericanacademyofophthalmologywheretheyhavewrittenthe
sequenceofdevelopmentofvariousstructures)

6) Regardinglens
AnteriorsutureinvertedYshaped
Suspensoryligamentsattachedtosulcus
indiabeticsincreasedsorbitol
aqueoushumorhasmoreascorbicacidthanlens(Correct,ReferanceJohnferris)

7) Whichisofneuroectodermalorigin
Sphincterpupillae(correct)

8) Whichnervepassesthrucavernoussinus:
Abducent

9) RegardingChoroid:
SuppliesRPE

10) Choriocapillaris:
Originatedirectlyfromophthalmicartery
Maximumatmacula(correct)

11) Reasonofmiosisiniridocyclitis
Waterlogging(imarked,,reference:Jogiwhereitswrittenthatthereasonofmiosisis
waterloggingandvasodilatation)
irritantscausecontractionofsphincterpupillae

12) Regardingvagusnerve
Passesthrumiddlecompartmentofjugularforamen(correct)
Givessensorysupplytomiddleear

13) Lensbestusedin:
Regularastigmatism(correct,ReferanceJogi)
Irregularastigmatism

14) Focallengthgiven0.75m.calculatePoweroflens.
power=1/fsoansweris1.33

15) Anteriorembryotoxon
Juvenilearcus

16) OcularmanifestationofSturgeWebberSyndrome
Glaucoma(Correct)
Retinalhemangioma

17) Mostrefractionby:
Anteriorsurfaceofcornea(correct)
Postsurfaceofcornea
Antsurfaceoflens

18) Firstsignofhypertensiveretinopathy:
Arteriosclerosis(correct)
microanerysms
Haemorrhages

19) InheritanceofColorblindness
Xlinkedrecessive

20) Mostcommontumorinchildren:
Capillaryhemangioma
Cavernoushemangioma
Wilmstumor(imarkedbutconfirmit)
Leukemiawasnotinoptions

21) ScenerioofAchondroplasia,inheritanceasked
AutosomalDominant

22) VerylongScenerioofDiGeorgeSyndrome

23) PathophysiologyofMyestheniagravis
Antireceptorantibodies

24) VitreousVolume:
4cc

25) RegardingYganglioniccells
Carrycolorcodedsignals
RelayinLayer1,2oflateralgeniculatebody(Correct.ReferenceJohnFerris)

26) Mosttoxiclocalanesthetic:
Bupivacaine(Correct,ReferanceAmericanAcadmeyofophthalmology)
Lignocaine
Alkane

27) Localanestheticcausingmostvasoconstriction:
Cocaine(correct)
Bupivacaine
Lignocaine

28) Lightreflexlostaccomodationpreserved.Lesionat
PretactalNucleus

29) Conjunctivalepithelium
StratifiedColumnar

30) RegardingConjunctiva
Gobletcellsscatteredinlaminapropriaofconjuctiva(Correct,referenceSnellanatomy)
Becomescontinuouswithbulbarfasicaclosetolimbus(itisattachedtobulbarfascia

3mmfromlimbusnotclosetolimbus)

31) Hypermetropia,whereimagewillbeformed?
Behindtheretina

32) Mechanismofactionoflocalanesthetics:
BindtoNachannelsinionizedform(correct)
BindtoNachannelsinunionizedform

33) Medialdeviationandextortionofeye,,lesionin?
Trochlearnerve(Correct)
abducentnerve

34) Pituatryadenoma:
bitemporalhemianopia

35) Whichdrugincreasesoutflowfromcanalofschlemm(Latanoprostnotinoptions)
Pilocarpine(imarked)
Timolol
Adrenaline
Phenylephrine

36) RegardingEdingerWestphalnucleuslesion:
Onlyaccomodationislost
Bothlightreflexandaccomodationarelost(Correct)

37) Vitreousisstronglyattachedto:
Vitreousbase(correct,ReferanceAAO)
vitreoretinalscar
Opticdisc

38) Drugofchoiceforaccomodativespasminchildren
Cyclopentolate(Correct)
Atropine
homatropine

39) Traumabytreebranchineyefollowedbypainlacrimationphotophobia,cause?
Fungalkeratitis(Correct,ReferanceJogi)
Type1hypersensitivity

40) ChildbornwithCataract.Mostlikelycause?
Rubellainfectioninmother

41) RightMCAinfarct.Whichiscorrect?
Aphasia
Paralysisofcontralaterallowerlimb
Droolingofsaliva
Homonymoushemianopia(correct,confirmedfromgoogle)

42) Suddenlossofaccomodation,OnCTscanwhereistheinfarct?
Midbrain(notsure)
Cerebrum

43) Secondarycartilagenousjoint:
Pubicsymphysis

44) LongscenerioofCushingsyndrome

45) RegardingBone:
Growsbyinterstitalgrowth
intramembranousossificationbycartilagemodelfirst
Osteoidistheunmineralizedbone(correct)

46) ScenerioofMixedconnectivetissuedisease,ANApositive.PolyarthritisRaynaurds
phenomenongiven,generalizedlymphadenopathy.Confirmatorytest?
AntiHIVab
RAfactor(correct)

47) Relationofophthalmicarterytoopticnerveinopticcanal?
Medially
Belowandlateral(Correct)

48) AquestionaboutVisualpathway.Correctanswerwaswhenlightisshowninonepupil
bothpupilsconstrict

49) H2O2foundin:
Peroxisomes(correct)
Mitochondria
Lysosomes

50) Regardinghistology
Gobletcellsreplacedbyclaracellsatterminalbronchioles

51) Nervesupplyoffibrousandparietalpericardium
Vagusnerve
Phrenicnerve(correct)

Intercostalnerve

52) Bestforstudyingcauseofrarediseases?
Casecontrol(imarked)
Casereportandstudy
Cohortstudy

53) Regardingcardiacreserve
%ageofcardiacoutputthatcanbeincreased(correct)
cardiacoutputdividedbybodysurfacearea

54) Acrosomalcapformedby:
Golgiapparatus(Correct)
Lysosomes

55) Basicdrugsbindsto:
Albumin
alphaglycoprotein(correct)

56) Superficialtemporalarteryaccompaniedby:
Auriculotemporalnerve(Correct)
Postauricularnerve

57) Whichofthefollowingcannotregenerate?
Lens(correct)
Kidneys
Cornea

58) Childwithhydrocephalusandabulgeinlumberregioncontainingnervoustissue.
Diagnosis?
Meningocele
Meningomyelocele(correct)
Meningoencephalocele

59) Manwithdryeyesanddrymouthwithoralulcerations
Sjogrensyndrome(correct)
MIkulizsyndrome

60) Dryeyes.Lesionat:
Greatersuperficialpetrosalnerve(correct)

61) Hydrocarboncauseswhichcarcinoma?
BronchogenicCA(correct)

Aplasticanemia

62) Whichismostlikelycarcinogen?
Asbestos(correct)
Forgototheroptions

63) PatientwithTB.worksinmine.Lesioninupperlobeoflung.Diagnosis?
Silicosis(correct)
Asbestosis

64) Lacrimalglandtype:
Serous(correct,ReferanceJohnferris)
Mucous
Mixed

65) Preganglioniccellbodiesofsphincterpupillae?
Ciliaryganglion
EdingerWestphalnucleus(Correct)

66) OxytocinandADHareproducedfrom
Hypothalamus(correct)
Pituitarygland
Adrenals

67) Posteriorpituitaryoriginatefrom:
Rathkespouch
Neuroectoderm(Correct)

68) Circadianrhythmby?
Suprachiasmaticnucleus(correct)
Arcuatenucleus

69) Boywithshortstatureandgenitalhypoplasia?Cause?
Craniopharyngioma(correct)

70) WhichisattachedtoWhitnalltubercle?
Aponeurosisoflevatorpalpabraesuperioris(correct,ReferanceSnellEye)
Lateralrectus

71) Atwhichplaceabducentnerveissusceptibletoinjury?
Petroustemporalbone
Cavernoussinus(Correct)

Clivus

72) Regardingvalsalvamanuver
Forcedinspirationagainstclosedglottis
NotassociatedwithBradycardia
Associatedwithtachycardia
Leadstofallinarterialpressure(Correct,,notsure)

73) Rapidbloodlossmostaffectedpartofkidneyis?
PCT(correct)
DCT
LoopofHenle

74) Farmerworkinginfieldsdehydrated,sodium120mmol/l,diagnosis?
Heatexhausation
Heatstroke
Excessivesweating(correct)

75) Somatosensorycortexsenddescendingfibresto?
Thalamusonly
Thalamusandspinalcord
Thalamusspinalcordanddorsalcolumn/medialleminiscus

76) Embolitobrain,whichtypeofnecrosiswilloccur?
Coagulative
Liquifactive(correct)
Casseous

77) Onequestionaboutmediancalculation

78) Causeoferrorinastudyrequiringlongtermfollowup?
Attritionbias(Correct,referancePark,Communitymedicine)
Lesscompliance
Faultytechniques

79) Defecationdueto?
Massmovement(correct)
Gastrocolicreflex

80) RegardingSchwlbesline?
AbruptdiscontinuationofDescmentsmembrane(correct)
inpostpartofcanalofschlemm

81) Regardingcentralretinalartery?
Noorbitalbranches
runsinopticnerveuptolaminacribrosa(correct)

82) Farmersprayingincrops,excessivesalivationfrothingfrommouthdiarrhea,most
obvioussigns?
Bilateralconstrictedpupils(Correct)
Mydriasis

83) Redgreenblindness,drugcausingit?
Ethambutol(correct)
Pyrazinamide
INH

84) Regardingmuscaraniccholinergicreceptors?
PresentatneuroeffectorjunctionofparasympatheticNS(Correct)
PresentatneuroeffectorjunctionofsympatheticNS
Atautonomicganglia

85) Miosisnotrespondingtoanydrug?
Cervicalsympatheticchaindamage

86) Whichmembranepresentbetweenretinaandchoriod?
Bruchsmembrane

87) %ageofocularinvolvementinleukemias?
40%(imarked,,butinreferencesitswrittenthatrangeinbetween3090%)
50%
60%
70%
80%

88) Trueaboutlipidlayeroftearfilm?
Preventsevaporationofoverlyingtearfilm
Lubricateseye(Correct..AiswrongbecausetearfilmisUNDERlipidlayernot
overlyinglipidlayer)

89) Trueaboutsympatheticactivation?
Relaxessmoothmuscleofbronchii(Correct)
Relaxesdilatormuscleofiris
Relaxessphincters

90) Regardingaccomodationtrueis?
Decreasedcurvatureoflens
ciliarymusclecontraction(correct)

91) Naturalantithromboticinbody?
Plasminogen(correct)
Heparin

92) Somatostatininhibitsinsulinby?
Paracrineaction(correct)
Autocrine
Holocrine

93) Tonicitynotequaltoplasma?
5%D/W(Correct)
N/S
Ringers
HumanAlbumin
1.4%Sodabicarb

94) Childwitheyetrauma,reducedvision,visionimprovedafterusingmedicineafter3
days?Diagnosis?
Subtotalhyphema(Correct,ReferanceAAO)
RD
Vitreoushemorrhage

95) Whathappensduringacclimatizationtohigheraltitude?
PulmVasodilation
increasedpulmventilation(Correct)
increasedoxygen

96) Femalewithprematureruptureofmembranes,after2daysstillbirth,examinationshows
absentvilliandneutrophilspresent.Reason?
Complement(imarked..notsure)
immunesystem
prostaglandins???
??

97) Hodgkinsdisease,bestprognosisin?
Lymphocytepredominance(correct)
lymphocytedepletion
Mixedcellularity

98) Pregnantfemaleirondeficiencyanemia,,ironstudiesshowed?
IncreasedTIBC(correct,referenceFirstaid)
increasedferritin
(serumtransferinnotinoptions)

99) Regardingazathioprine?
Supressesbothcellularandhumoralimmunity
inhibitsdihydrofolate
MetabolizedbyXanthineoxidase(correct,confirmedfromGoogle)

100) WhichhormoneactsbyPhospholipaseC?
Vasopressininducedvasoconstriction(correct)
waterreabsorptionbyADH

101) HighestPco2in?
Expiredair
Pulmonaryartery(correct)

102) Virusactsby?
alteringproteinsynthesis

103) DifferencebetweenbenignandmalignantCA?
Metastasis(correct)
invasion
Atypia

104) DiagnosticcriteriaofCAonmicroscope??
Pleomorphism(correct)
invasiontoadjacentstructure

105) Gastriculcerleadingtoperforationandirritatingthegastricperitoneum,paincarried
by?
Greatorsplanchnicnerve
intercostalnerves(Correct,referenceSnellanatomy)

106) Leftadrenalveindrainsinto?
Leftrenalvein(correct)
IVC

107) Valvelessvein?
SVC(correct)
IVC
Thoracicduct

Azygousvein

108) Giantcellvasculitisassociatedwith?
Polymyalgiarheumatica

109) Whichenzymetransferaradicalfromonecompdtoother?
Transferase(imarked)
Oxidoreductase

110) Nolymphnodules?
Spleen
Lymphnodes
Thymus(correct)

111) Impressionaboveleftlungrootby?
Archofaorta(correct)
azygousvein

112) Flaccidparalysisandfoodpoisoning.diagnosis?
Botulism

113) Increasedtoxicityofaminoglycosidesinelderlybecause?
Decreasedrenalexcretion

114) Scantybarrbodyin?
Turner(correct)
Klinefelter

115) Duetorapidventricularfilllingwhichheardsound?
s1
s2
s3(correct)
s4

116) Regardingtransplantfromidenticaltwins?
Notrejectedevenwithoutimmunosupression(imarked)
RejectedslowlyduetominorHLA

117) Confidentialitycanbebreechedwhen?
Patientallowsuto(correct)
Insuranceclaimnotinoptions

118) RegardingC1vertebrae?
Hasnobody(correct)
Longspinousprocess

119) Pregnantladywithobstructivejaundice.investigationofchoice?
GGT(correct)
Alkalinephosphatase

120) Autoregulationinbrainby
pco2

121) Pyogenicperitonitisby
Ecoli
Bacteriods(correct)

122) Mostcommongeneticdisorder?
Autdominant
Multifactorial(correct)

123) Hypermagnesimialeadsto?
decreasedAchrelease(Correct)
increasedAchrelease

124) Muscleinvolvedinsadnessexpression?
Platysma

125) AnalogousofSchwanncellsinCNS?
Oligodendrocytes

126) ScenerioofScleroderma

127) WhichoneinhibitsProstaglandinsynthesis?
Aspirin
Corticosteroids?(imarked)

128) Preganglionicautonomicfibres?
Bfibres

129) Irreversiblesignofcellularinjury?
Contractionbands(correct)
Cellswelling
Decreasedglycolysis

130) Myocardialbloodflowmaximallyincreasedby?
Sympatheticactivation??
Myocardialhyperoxia

131) Serotoninmaximallyproducedby?
argentaffincellsofGIT(Correct)
Platlets

132) Middlemeningealarterybleed
B/wCalvariaanddura(correct)
b/w2layersofduramatter

133) Verysmallparticlesinlungsremovedby?
Cilia
phagocytosis(correct)

134) SimpleDiffusiondependsupon
Surfacearea(correct)
SaturationkineticsNernstpotentialsomethinglikethat

135) Osmosisdependsupon?
No.ofsoluteparticles(itsacolligativeproperty)

136) Lesionatcavernoussinus,whichwillnotbeaffected?
Occulomotornerve
Abducent
Pituartygland
Olfactorynerve(correct)

137) 1Lbloodlostwhichwilltakelongesttimetoreturntonormal?
BP
Renin
Aldosteron
RBCs(imarked)

138) 70%ofoxygenextractedbywhatinrestingcondition?
Heart(correctaccordingtoRabiaAliMcq)
Brain
Liver

139) Bloodgroupantigensare?
Glycoproteins

140) AdultPCKDassociatedwith?
Cerebralhaemorrhage(Correct)
Renalfailure

141) Scenerioofmegaloblasticanemianautoimmunedisease..Investigation?
Serumb12levels
intrinsicfactorantibodies(correct)

142) Socialsupportinoursetupby?
Family(correct)
NGOs

143) Ironstoredinparenchymainformof?
Hemosidren
Transferrin
Ferritin(correct)

Regards
Kindlyrememberinprayers

CompiledbyDrKhadijaZahid

RADIOLOGY 1st SHIFT 5 SEP 2016


1. Earliest sign of Vitamin A deficiency is
NIGHT BLINDNESS
2. Young boy with pain less swelling of right orbit. CPSP Ques.
OPTIC NERVE GLIOMA
3. Hypermagnesemia causes
a. decreased Ach release = ans
b. increased Ach release
c. tetany
d. arrhythmia
4. Origin of thyroid follicular cell is
ENDODERM
5. Epithelium of Uterus and Vagina is derived from
Endoderm and Mesoderm
6. A woman had fallopian tube ligature, she is found to be in shock after 6 hours, Laparotomy is
done and intraperitoneal hemorrhage is found. bleeding is most likely from branches of
a. uterine artery
b. common iliac vessels
c. external iliac vessels
d. aorta
7. A man walks 100 yards and felts pain in calf, relieved after stopping, artery involved is
POPLITEAL ARTERY
8. Left renal vein is posterior to
a. head of pancreas
b. 3rd part of duodenum
c. aorta
d. IVC
e. Portal vein
9. Nick during surgery on right side of hepatoduodenal ligament will cause damage to
a. Cystic duct
b. IVC
c. hepatic veins
d. hepatic artery
e. portal Vein = Ans

10. Mass at portal hepatis will compress


same options as Quest 9
ANs is Portal Vein as No CBD was mentioned
11. Regarding Cephalic vein
a. starts from medial dorsal surface of hand
b. connects with basilic vein in elbow.
c. Runs in deltopectoral groove (ANS)
d. pierces Clavipectoral fascia
12. which cranial nerve nuclei lies lateral to rhomboid fossa/sulcus limitans
a. dorsal vagal nucleus = Ans
b. CN 6
c. CN 3
d. CN 11
e. CN 9
13. Posterior most fibers in posterior part of the rim of internal capsule contain
a. optic radiations
b. corticospinal fibers of upper limb
c. CS fibers of lower limb
d. frontopontine fibers? (I marked)
e. thalamocortical tract
14. Man during running develops aching pain in ankle, next days when he wakes up he founds
ecchymosed around ankle joint, he can stand on tip toes but it hurts.
ANS = RUPTURED PLANTARIS TENDON
15. structure anterior to pancreas, these were 5 options
A. hepatoduodenal ligament
b. left colic
c. portal vein
d. superior mesenteric vessels = I marked
e. splenic vessels
16. vessel dilated most likely in portal HTN
a. Left colic vein
b. inf. epigastric vein
rest options were of systemic vessels I think best is inf. epigastric as left colic is br. of IMV
though may be dilated but most obvious are inf epigastric

17. Neurons of vestibular nuclei ends at


a. cortex
b. dentate nucleus
c. granular layer of cerebellum = AnS
d. cochlear nucleus
18. Irreversible change in MI is
Ans = Contraction bands
19. Feathery appearance in
Jejunum Ans
20. Child of 2 years age presents in opd with Fever, diarrhea and pallor for 2 days. Hob 5.6%, MCV
107 MCHC inc. Inv of choice is
a. transcobalamin levels
b. intrinsic factor antibodies assay
c. serum b12 level = ans
d. rbc folate
21. Right testicular CA 1st metastasizes to
A. Aortocavolumbar nodes
b. renal hilar nodes
c. sup. inguinal
d. common iliac
22. Extra embryonic coelom is derived from
EPIBLAST = ANS
source BRS & High yield Embryo
23. True about Trigeminal ganglion
A. lies in Ant. cranial fossa near crista galli
B. lies in apex of petrous part of temporal bone in middle cranial fossa = ANS
c. covered in ganglion
d. bathes in CSF
24. Young adult involved in RTA, got injury to middle cranial fossa, bleeding from internal ear. After
recovery presents with loss of tear formation. Injury to which structure during RTA caused this
a. Greater superficial petrosal nerve = ANS
b. deep petrosal nerve
c. interaortocaval plexus
d. cilliary ganglion
e. carotid plexus

25. A man presents with gradual loss of vision, radiological studies show growing clot in a vessel, the
most likely involved vessel is
a. vertebral artery
b. basilar artery
c. MCA
d. PCA = ANS ??
e. posterior communicating artery
26. A woman presents with bleeding with superficial cuts. O/Inv. only BT time is prolonged. defect is
due to
a. thrombocytopenia = ANS
b. factor 8 def
c. factor 9 def
27. Regarding most correct statement about bundle of HIS
a. only pathway conducting impulses from atria to ventricles
b. situated in membranous part of IV septum near atrioventricular junction
rest options were wrong
28. most prominent structure during IVU of kidney is
CALYCES = Ans
29. Amino glycoside half life is increased in Old age patients due to
a. hepatic metabolism is decreased
b. dec renal function = ANS
c. dec Vol of distribution
30. A patient comes with swelling of big toe on right side of foot. no pain was mentioned in
question directly asked inv.
a. serum uric acid
b. serum urea and uric acid
c. microscopy of joint fluid
d. urine uric acid
31. Clara cells are present in
terminal Bronchioles = Ans
32. Forehead is formed from
A. Fronto Nasal Process
B. maxillary process
c. palato nasal process

33. A footballer gets injured during match, presents in ER with limb instability. O/E there is swelling
in pre-patellar region and his Tibia is drawn forward on Femur. The structure most likely injured
is
A. Anterior Cruciate Ligament = Ans
34. 8 patients were reported in a case study, their age was 20,25,25,30,40,20,30. The median of
values is
27.5 = ANS
median is calculated with values put in ascending order and if odd no. then the central one is
median. If even no. then middle two values average, in this case( 25+30)/2 is ans
35. Regarding anatomy of Inguinal Canal
a. lies from ASIS to pubic tubercle
b. Superior boundary formed from conjoint tendon = ANS
36. A study in which diseased people are compared to non diseased healthy people is called
a. cross sectional
b. cohort
c. case control
d. RCT

37. In growing embryo, the mesothelium covering the Lung forms


a. pleuroperitoneal membrane
b. pleuropericardial membrane
c. parietal pleura
d. visceral layer of pleura
e. fibrous pericardium
38. Food Intake/reflex
a. is promoted if hunger center is damaged
b. is inhibited if satiety center is damaged
c. inhibited by leptins secreted from adipocytes
d. inhibited by leptins secreted from myocytes
39. A patient presents with cough. CXR shows B/L peri-hilar infiltrates. Biopsy of skin lesions shows
granulomatous inflammation. Diagnosis most likely is
a. silicosis
b. TB
c. sarcoidosis = ANS
d. asbestosis
e. histoplasmosis

40. Extensor muscles of arms are supplied by


Posterior Cord = ANS
41. In fracture of hamate, which is involved?
a. ulnar artery
b. ulnar nerve
rest were wrong
42. Man with lymphoma was being treated with anti cancer drugs, biopsy of a lymph nodes shows
clumping of nuclear material and fragmentation. The most likely mechanism is
Apoptosis = ans
43. In infectious mononucleosis organ at risk of rupture is
SPLEEN = Ans
44. P. Malaria transmits to human in form of
Sporozoite = Ans
45. A male from Baluchistan presents with lethargy, joint pains and leg ulcers. Diagnosis will most
likely
A. sickle cell
b. thalassemia major
c. HbC
46. A child presents with pallor, lethargy and dec activity. O/I shows dec Hb dec MCV and Bone
changes are prominent on Face
a. thalassemia major
b. sickle cell disease
c. thalassemia alpha
47. Regarding RBC antigen on surface
A. glycolipids
B. Glycoprotein = Ans
mot written in Ganong 24th ed or Guyton 12th ed. Searched from Google still a disputed matter
48. Regarding ankle joint
a. surface covered from fibro cartilage
b. deltoid ligament on MALLEOLAR SIDE (medial) was not written
c. superficial peroneal nerve is only supply to it
d. talofibular ligament is on medial side
49. You are allowed to breach patients confidentiality when
a. patient allows to do so
b. insurance claim

50. A patient presents with central chest pain for 4-6 hours. Pain is radiating to arm and jaw. Best
marker to confirm MI at this stage is
a. CPK
b. CPK MB = Ans (as no trop T mentioned)
c. LDH
51. A patient had heart-lung transplantation and was on immunosuppressive drugs now presents
with high grade fever, neck rigidity +ve kernig. CSF studies show an organism with clear
cytoplasm with a halo around it.
a. CRYPTOCOCCUS = Ans
b. EBV
c. staph aureus
d. klebsiella
e. CMV
52. In Shock which is the most important mechanism which conserves the organism as a whole
a. baroreceptor
b. sympathoadrenergic system = Ans
c. CNS ischemic response
Option B is best bcz it conserves perfusion to vital organs while CNS ischemic response comes at
end stage when perfusion to brain is low, it causes widespread vasoconstriction, which causes
organ failure.
53. Remnant of Umbilical Vein is
a. falciform ligament
b. ligament of teres = Ans
Teres is a part of falciform and is actually the remnant
54. Patient had hypovolemic shock, presents with signs of Acute Renal failure. Most likely damage
to which part of the nephron is greatest
a. PCT = Ans
b. descending
c. ascending
d. DCT
e. Collecting duct
55. A school going boy develops Right sided flank swelling and lumbar pain, US shows
Hydronephrosis of the right kidney while the other is normal. Which of the below is most likely
the cause of it
a. Urethral stricture
b. ureteral stricture
c. posterior ureteral valves = I marked as no other was found to be more appropriate
d. vesico ureteric reflux

56. A patient had an Accident with fracture of surgical neck of humerus. Most likely involved nerve
is
a. axillary = ANS
b. radial
all nerves options
57. Lymph node of inner breast drains into all except
a. inf phrenic nodes
b. supraclavicular nodes
c. internal thoracic nodes
d. pectoral nodes = ANS
58. Most common fracture of femur at age of 60 is
A. femur head fracture
b. intra capsular fracture
c. neck of femur fracture = Ans
d. shaft fracture
59. Epidural Space Ends at
a. S2 = Ans
60. Left gastric artery is a branch of
a. Celiac Artery = ans
61. Structure accompanying esophagus in diaphragm is
a. left vagus
b. right vagus/vagal trunk = Ans
c. phrenic nerves
d. azygous vein
62. Inhibitors of Prostaglandins are
A. Corticosteroids = Ans
b. aspirin
63. Hall mark of Adreno cortical insufficiency is
a. Hyponatremia = Ans
b. hypokalemia
c. metabolic alkalosis
d. hypocalcemia
64. Fast Pain travels by
A. A delta fibers = Ans
65. Sympathetic nerves ending at Inferior hypo gastric plexus have root value of
A. L1-L2 = Ans

66. Detrusor muscle of bladder supplied by


A. pelvic Splanchnic nerves (parasympathetic)
67. Pectoral group of axillary nodes drain
a. Upper half of trunk
b. Upper half of upper limb
c. Major part of the breast = Ans
d. Lower half of trunk
68. Regarding Vagus Nerve
a. Exits through passing middle part of jugular canal = Ans
b. Exits thorax by passing with Aorta
other options cant recall best may be A
69. Fragile X syndrome is
a. Trinucleotide repeat disorder = Ans
70. Most common cause of congenital inherited defects
a. Multifactorial = I marked
b. Autosomal dominant
71. A patient presents in OPD with vesicular eruptive rash which is starting from left temporal side
and ends near medial side of left eye. Rash is painful, diagnosis of Shingles is made. Which of the
following nerve may be involved
a. Ophthalmic nerve
b. Superior orbital nerve
c. Supra trochlear nerve
d. Maxillary
e. Mandibular nerve
72. Acrosome is formed by
a. Golgi bodies = Ans
b. Endoplasmic reticulum = I marked bcz I forgot it despite reading :D
73. External urethral sphincter related to prostatic:
a. Apex
b. Base
c. Posterior
d. anterior side
the question doesnt mentioned prostate or prostatic urethra, it was written Prostatic in Q.
74. Small particles are cleared by
A. Cilia in terminal bronchioles = Ans

75. Highest PCO2is in which vessel


a. Pulmonary artery = I marked
b. IVC = should be this bcz O2 consumption is highest in lower body and low in head.
c. SVC
76. Not isotonic to plasma is
a. 5% dextrose water
77. Lumbar triangle anteriorly bound by
a. Posterior border of External Oblique = Ans (SNELL)
78. Ulcer 2nd part of duodenum at posterior side artery eroded is
a. Gastro duodenal artery = ans
79. Slowest growing tumor of thyroid
A. Papillary carcinoma = ans
80. Commonest dislocation of TMJ is
A. Anterior = ans
81. A man has short trunk and longer arms, this disease is ________ .
a. Autosomal dominant disorder = Ans
82. Type 3 reaction is
a. Arthus reaction = ans
83. Myelin formation in CNS is from
a. Oligodendrocytes
I HAD 2 Questions of this in paper 2, straight forward Q & A
84. Mandibular branch of trigeminal nerve exits cranial cavity through
a. Forman ovale = ans
85. Heme in blood binds with
a. Haptoglobulin = ans
b. Hemopexin
86. Cerebral blood flow is regulated by
a. PCO2 in venous blood = Ans
b. pH of CSF
87. Eparterial bronchus is
a. Rt superior = ans

88. Epidural sinus in spinal canal


a. Longitudinal venous plexus = I marked other options dont remember
b. CSF
c. Is between Dura and arachnoids space
89. Most common congenital anomaly of head is
a. Cleft lip & palate = ans
90. A woman head Radical mastectomy after recovery unable to raise arm above her head,
structure damage is
a. Long thoracic nerve = ans
91. Anterior relation of left kidney which is directly in contact with kidney without any membrane
a. Lesser sac
b. Pancreas = ans
c. Duodenum
d. Left colic flexure
92. Structure passing above lower head of Pancreas
a. SMA = ans
93. ICAM and VCAM are.
a. adhesion molecules = ans
94. Derivative of 2nd arch is
a. Stylohyoid = ans
rest were 1st arch derivatives
95. Adverse effect in long term use of glucocorticoids is
a. Osteoporosis = ans
rest were irrelevant
96. Branches of brachiocephalic trunk are.
a. Rt common carotid and Rt Subclavian
97. If LCX is occluded what will be the effect
a. Posterior of left ventricle be more affected = ans
b. SA nodal block will occur
c. Post interventricular septum 1/3 will be infarcted
98. SA node is pacemaker of heart because
A. It lies at the bottom of atrioventricular junction
b. It lies at the top of RT atrium = ans
c. It has a plateau
d. Its fibers have fastest conduction

99. Heinz bodies are present in


G6PD
100. 2 year old child was playing with toys when he suddenly become suffocated and cyanosed for
few seconds. Mother took him a hospital on CXR he had ate a small coin, where will it be lodged
most likely
A. Rt lower basal segment
101. A young child inhaled a peanut where it will be lodged most likely
a. Lower lobe of right lung
these were 2 diff questions with same answers both in paper 2
102.Intrinsic factor is secreted from
Gastric fundus
103.Virus causes
Alteration of protein synthesis
104.Moral support to patient in our society comes from
Family
105.A patient is binge drinker of Alcohol presents to ER with C/O RT side hypochondrial pain. US
shows fatty liver, Biopsy shows councilman bodies in hepatocytes.
Apoptosis
106.Feature of Atlas is
Absent Body
107.A patient is being evaluated for TB. The best investigation which will diagnose TB is
a. Caseous necrosis on Biopsy
b. Epitheloid Cells
c. Demonstration of Acid fast bacilli on ZN stain
108.Middle meningeal artery
Enters cranial Cavity through foramen Spinosum
109.What parameter is increased in Pregnant women
a. TIBC = ans
b. Serum ferritin
c. PCV
d. Hb
110.Nerve supply to pericardium is from
Ans = phrenic Nerve

111.Highest energy compound is


a. Fats
112.Pubic bones join anteriorly by
a. Secondary cartilaginous joints
113.Vein having no valves is
Ans= SVC
114.Following vein is a tributary of Portal vein
a. Superior rectal vein = ans
b. Hemiazygous vein
c. Inf epigastric vein
d. Left renal vein
e. Lumbar veins
115.A lady has difficulty in standing from a sitting position but can walk normally. Muscle involved is
Ans. Gluteus Maximus
116.A detached embolus from deep veins of leg will go first to
a. IVC
b. Rt Atrium
c. Lt ventricle
d. Pulmonary veins
e. Pulmonary artery = Ans
117.Defect of RBCs in Hereditary Spherocytosis
Ans. Cytoskeleton abnormality
118.Mixed venous blood sample can be obtained from
a. SVC
b. IVC
c. Rt atrium
d. Rt ventricle
e. Pulmonary arteries
119.Long term glucose control can be best detected by
Ans. HbA1C
120.Glycocalyx is
a. Polysaccharide moiety on bacterial cell wall
b. Carbohydrate chain on cell wall
rest options didnt remember

121.Basic drug binds with


ans. Alpha glycoprotein
122.Thyroid drains to
Ans = deep cervical nodes
123.Pregnant women with viral hepatitis. Best inv. to detect is
Ans. GGT
124.1/3rd of TBW is
Ans. ECF
125.Which hormone level will fall after drinking water
a. ADH
b. Angiotensin
126.Maximum blood pressure is at
a. Renal artery
127.Most common cause of K shift from ICF to ECF
Ans. Vigorous exercise
128.Glucose is main source for
ans. Neurons
129.Oxytocin and ADH originate in
a. Hypothalamus
130.Most common association of ADPKD is
a. Renal failure
b. Cerebral hemorrhage
131.Muscle constructing causes sadness
a. Platysma
132.Clostridium botulinum causes
a. Flaccid paralysis
133.Nerve accompanying superficial temporal artery is
Auriculo-Temporal Artery
134.Sarcoma differs from Carcinoma in
Increased Vascularity = ans

135.Patient on table heat loss mainly by


a. Conduction = ans (new ppl read this topic from ganong or Guyton)
b. Radiation
c. convection
136.Acidic urine in Tubules will cause
a. increased secretion of CLb. Decreased HCO3c. Dec K+secretion
d. Reduced H+ reabsorption ( I marked).
e. Aldosterone inc or dec dont remember
137.Baby born with congenital cataract, mother had most likely which intrauterine infection
A. Rubella
138.Left adrenal vein drains to
left renal vein = Ans
139.Lower Esophageal Veins drain to
A. Portal Vein
140.Left lung has groove for
a. Ascending aorta
b. Descending aorta
c. Arch of aorta = ans
d. Azygous vein
141.Deep vein thrombus 1st lodges in
a. Branches of pulmonary artery = ans
b. Rt atrium
c. Rt. Ventricle
d. IVC
142.Patient had TB, developed pleural effusion. Needle inserted will be along
upper border of lower rib = ans
143.Cause of pyogenic peritonitis is
a. E. Coli = ans
b. Bacteriodes
c. Klebsiella
d. Staph aureus
144.Metaplasia is
one cell type replaces other with functional change = ans

145.A patient had a coronary bypass developed A-fib now presents in ER with signs of stroke. Brains
shows which type of necrosis
a. Liquefactive = ans
146.HIV affects
CD-4 Cells T = ans
147.Labor working in hot weather collapses. Most likely abnormality found in him will be
a. Hyponatremia = ans
b. Hypervolemia
c. Hyperkalemia
d. Hypercalcemia
148.Carcinoma compared to adenoma has
a. Capsule
b. Invasion = ans
c. No loss of differentiation
d. Vascularity
149.TB resists immunity. In this case Antibodies are found
a. Attached to cell
b. Bound to organism
some other options were too, cant recall
150.Iron is stored in parenchyma under normal conditions is
Ferritin = Ans
151.Regarding saliva
a. Has inc Na as compared to plasma
b. Low K & HCO3 in comparison to plasma
c. Hypo tonicity is more than plasma = ans
152.Osmotic pressure depends upon
a. No of particles = ans
b. Diffusion potential
c. Charge on membrane on either side
153.Diffusion potential depends upon
a. Surface area of membrane = ans
b. Charge on membrane
c. Charge on either side
d. From low conc. to high conc.

154.From 3rd to 9th month of pregnancy, source of estrogen is


a. Corpus luteum
b. Granulosa cells
c. Placenta = ans
155.Strength provided to wound during healing is by
a. Collagen 1
b. Collagen 3 = ans
156.Granulosa cells are found in
a. Corpus luteum
b. Ground substance
c. Graffian follicles = I marked this or resembling one to this opt
d. Follicular epithelium
157.Esophageal constriction is by
a. Left atrium = ans
158.Regarding Rt Atrium
a. Forms lower border of sternocostal surface = ans
b. Is the posterior most structure
c. IVC ans SVC junction is visible at outer surface
d. Is supplied by LCX
159.Regarding bone formation
A. Newly laid bone is in the form of osteon = I marked other options not remember
160.Vibrations during Rapid Ventricular filling produces
A. 3rd heart sounds = ans
b. 4th heart sound
161.TPR is directly related to
a. Capacitance
b. HR
c. Vasomotor tone = ans
162.Exposure to hydro-carbons cause
a. Acute non lymphocytic leukemia
b. CML
c. AML
d. Liver cancer
163.Cause of mesothelioma is
Asbestos = ans

164.Sub acute bacterial endocarditis is caused by


Ans. Strep viridians
165.Cornayae diphtheria mode of transmission is
a. Oro-fecal
b. Respiratory droplets = ans
c. Skin contact
d. From spores
166.Inhibin is produced by
a. Sertoli cells =ans
167.Lymphatic nodules absent in
A. Thymus = ans
B. Spleen
c. Payers patches
168.Rapidly adopting fibers are.
a. Pacinian = ans
b. Meissners
c. Merkel
d. Ruffini
e. Free nerve endings
169.Regarding vagus nerve in GIT
a. Stimulation causes relaxation of ano-rectal sphincter
b. Contraction of longitudinal muscle group for peristalsis = ans
c. Decreased acid secretion
d. Constriction of oddi sphincter
170.In case of Anterior duodenal perforation, fluid will go to
a. Left paracolic gutter
b. Rt sub hepatic space
c. Rt subphrenic space
d. Rt Para colic gutter = ans
171.In case of posterior perforation of 1st part of duodenum, fluid will be collected into
a. Lesser sac
b. Rt subphrenic space = I marked
c. Rt Sub hepatic space
d. Rt paracolic gutter

172.Antibodies are formed by


Ans. Plasma cells
173.Glucocorticoids cause
a. Increased production of protein
b. Hepatic gluconeogenesis = ans
c. Dec amino acid metabolism
174.Which hormone acts through phospholipase C
a. Hormone causing increased blood flow in penis
b. Causing water reabsorption in kidneys Adh V2
c. Constriction of vasculature by ADH V1 = ans
d. ANP
e. Angiotensin causing vasoconstriction
175.Sensory supply to skin over lower half of deltoid is supplied by
Ans. Axillary nerve
176.Preganglionic autonomic fibers are
Ans. B fibers
177.Cells which are enlarged in adenoma of parathyroid
a. Chief cells = ans
b. Oxyphil cells
c. Both
178.Genital branch of genito-femoral nerve
a. Passes through deep inguinal ring = may be
b. Gives branches to anterior scrotal wall = I marked B = cant recall other options
179.Characteristic of IgM
Ans. Is a large molecule (pentamer)
180.Highest triglycerides % is in
Ans. Chylomicrons
181.Efficiency of a drug is
a. Efficacy = ans
b. Potency
182.Tension in muscle is sensed by
Ans. Golgi tendon

183.A patient presents to you in emergency. His pH is 6.95 HCO3 4, glucose 500mg/dl. You
administer ______ insulin
Ans. Regular insulin (couldnt remember was this question in paper or not)
184.Isthmus of uterine tubes is
A. Below the fundus
b. Is the narrowest part of tube = ans (THIS ONE TOO I DONT REMEMBER it was in paper or not)
185.Genioglossus supplied by
Ans. C1 fibers through hypoglossal nerve
186.Chondorcytes change into osteocytes
A. Thyroxin
b. Somatotrophs = ans
187.Patient with lower backache and pain radiating to posterior thigh. The most likely cause is
A. Disc herniation at L4-L5
b. Disc herniation at L5-S1 = ans
Couldnt recall other Questions, 183/184 were or were not in the paper but rest of the questions were in
1st shift Radiology 5th sep.
After compiling this I pray we all may pass. Amen
Remember in my prayers. If you want to add or correct any question, please message/inbox me, itll be
appreciated.
Regards: Dr. Muhammad Qasim.

RADIOLOGY 1st SHIFT 5 SEP 2016


1. Earliest sign of Vitamin A deficiency is
NIGHT BLINDNESS
2. Young boy with pain less swelling of right orbit. CPSP Ques.
OPTIC NERVE GLIOMA
3. Hypermagnesemia causes
a. decreased Ach release = ans
b. increased Ach release
c. tetany
d. arrhythmia
4. Origin of thyroid follicular cell is
ENDODERM
5. Epithelium of Uterus and Vagina is derived from
Endoderm and Mesoderm
6. A woman had fallopian tube ligature, she is found to be in shock after 6 hours, Laparotomy is
done and intraperitoneal hemorrhage is found. bleeding is most likely from branches of
a. uterine artery = ans
b. common iliac vessels
c. external iliac vessels
d. aorta
7. A man walks 100 yards and felts pain in calf, relieved after stopping, artery involved is
POPLITEAL ARTERY
8. Left renal vein is posterior to
a. head of pancreas
b. 3rd part of duodenum = I marked
c. aorta
d. IVC
e. Portal vein
9. Nick during surgery on right side of hepatoduodenal ligament will cause damage to
a. Common bile duct
b. IVC
c. hepatic veins
d. hepatic artery = Ans
e. portal Vein

10. Mass at portal hepatis will compress


same options as Quest 9 but CBD was replaced by cystic duct option
ANs is Portal Vein as No CBD was mentioned
11. Regarding Cephalic vein
a. starts from medial dorsal surface of hand
b. connects with basilic vein in elbow.
c. Runs in deltopectoral groove (ANS)
d. pierces Clavipectoral fascia
12. which cranial nerve nuclei lies lateral to rhomboid fossa/sulcus limitans
a. dorsal vagal nucleus = Ans
b. CN 6
c. CN 3
d. CN 11
e. CN 9
13. Posterior most fibers in posterior part of the rim of internal capsule contain
a. optic radiations
b. corticospinal fibers of upper limb
c. CS fibers of lower limb
d. frontopontine fibers? (I marked)
e. thalamocortical tract
14. Man during running develops aching pain in ankle, next days when he wakes up he founds
ecchymosed around ankle joint, he can stand on tip toes but it hurts.
ANS = RUPTURED PLANTARIS TENDON
15. structure anterior to pancreas, these were 5 options
A. hepatoduodenal ligament
b. left colic
c. portal vein
d. superior mesenteric vessels = I marked
e. splenic vessels
16. vessel dilated most likely in portal HTN
a. Left colic vein
b. inf. epigastric vein = ans
rest options were of systemic vessels I think best is inf. epigastric as left colic is br. of IMV
though may be dilated but most obvious are inf epigastric

17. Neurons of vestibular nuclei ends at


a. cortex
b. dentate nucleus
c. granular layer of cerebellum = AnS
d. cochlear nucleus
18. Irreversible change in MI is
Ans = Contraction bands
19. Feathery appearance in
Jejunum Ans
20. Child of 2 years age presents in opd with Fever, diarrhea and pallor for 2 days. Hob 5.6%, MCV
107 MCHC inc. Inv of choice is
a. transcobalamin levels
b. intrinsic factor antibodies assay
c. serum b12 level = ans
d. rbc folate
21. Right testicular CA 1st metastasizes to
A. Aortocavolumbar nodes = Ans
b. renal hilar nodes
c. sup. inguinal
d. common iliac
22. Extra embryonic coelom is derived from
EPIBLAST = ANS
source BRS & High yield Embryo
23. True about Trigeminal ganglion
A. lies in Ant. cranial fossa near crista galli
B. lies in apex of petrous part of temporal bone in middle cranial fossa = ANS
c. covered in ganglion
d. bathes in CSF
24. Young adult involved in RTA, got injury to middle cranial fossa, bleeding from internal ear. After
recovery presents with loss of tear formation. Injury to which structure during RTA caused this
a. Greater superficial petrosal nerve = ANS
b. deep petrosal nerve
c. interaortocaval plexus
d. cilliary ganglion
e. carotid plexus

25. A man presents with gradual loss of vision, radiological studies show growing clot in a vessel, the
most likely involved vessel is
a. vertebral artery
b. basilar artery
c. MCA
d. PCA = ANS ??
e. posterior communicating artery
26. A woman presents with bleeding with superficial cuts. O/Inv. only BT time is prolonged. defect is
due to
a. thrombocytopenia = ANS
b. factor 8 def
c. factor 9 def
27. Regarding most correct statement about bundle of HIS
a. only pathway conducting impulses from atria to ventricles = I marked
b. situated in membranous part of IV septum near atrioventricular junction
rest options were wrong
28. most prominent structure during IVU of kidney is
CALYCES = Ans
29. Amino glycoside half life is increased in Old age patients due to
a. hepatic metabolism is decreased
b. dec renal function = ANS
c. dec Vol of distribution
30. A patient comes with swelling of big toe on right side of foot. no pain was mentioned in
question directly asked inv.
a. serum uric acid
b. serum urea and uric acid
c. microscopy of joint fluid = I marked
d. urine uric acid
31. Clara cells are present in
terminal Bronchioles = Ans
32. Forehead is formed from
A. Fronto Nasal Process = ans
B. maxillary process
c. palato nasal process

33. A footballer gets injured during match, presents in ER with limb instability. O/E there is swelling
in pre-patellar region and his Tibia is drawn forward on Femur. The structure most likely injured
is
A. Anterior Cruciate Ligament = Ans
34. 8 patients were reported in a case study, their age was 20,25,25,30,40,20,30. The median of
values is
27.5 = ANS
median is calculated with values put in ascending order and if odd no. then the central one is
median. If even no. then middle two values average, in this case( 25+30)/2 is ans
35. Regarding anatomy of Inguinal Canal
a. lies from ASIS to pubic tubercle
b. Superior boundary formed from conjoint tendon = ANS
36. A study in which diseased people are compared to non diseased healthy people is called
a. cross sectional = I marked
b. cohort
c. case control
d. RCT

37. In growing embryo, the mesothelium covering the Lung forms


a. pleuroperitoneal membrane
b. pleuropericardial membrane
c. parietal pleura
d. visceral layer of pleura = ans
e. fibrous pericardium
38. Food Intake/reflex
a. is promoted if hunger center is damaged
b. is inhibited if satiety center is damaged
c. inhibited by leptins secreted from adipocytes = ans
d. inhibited by leptins secreted from myocytes
39. A patient presents with cough. CXR shows B/L peri-hilar infiltrates. Biopsy of skin lesions shows
granulomatous inflammation. Diagnosis most likely is
a. silicosis
b. TB
c. sarcoidosis = ANS
d. asbestosis
e. histoplasmosis

40. Extensor muscles of arms are supplied by


Posterior Cord = ANS
41. In fracture of hamate, which is involved?
a. ulnar artery
b. ulnar nerve = ANS
rest were wrong
42. Man with lymphoma was being treated with anti cancer drugs, biopsy of a lymph nodes shows
clumping of nuclear material and fragmentation. The most likely mechanism is
Apoptosis = ans
43. In infectious mononucleosis organ at risk of rupture is
SPLEEN = Ans
44. P. Malaria transmits to human in form of
Sporozoite = Ans
45. A male from Baluchistan presents with lethargy, joint pains and leg ulcers. Diagnosis will most
likely
A. sickle cell = Ans
b. thalassemia major
c. HbC
46. A child presents with pallor, lethargy and dec activity. O/I shows dec Hb dec MCV and Bone
changes are prominent on Face
a. thalassemia major = Ans
b. sickle cell disease
c. thalassemia alpha
47. Regarding RBC antigen on surface
A. glycolipids
B. Glycoprotein = Ans
mot written in Ganong 24th ed. or Guyton 12th ed. Searched from Google still a disputed matter
48. Regarding ankle joint
a. surface covered from fibro cartilage = I marked, but it is wrong
b. deltoid ligament on MALLEOLAR SIDE (medial) was not written
c. superficial peroneal nerve is only supply to it
d. talofibular ligament is on medial side
49. You are allowed to breach patients confidentiality when
a. patient allows to do so = ans
b. insurance claim

50. A patient presents with central chest pain for 4-6 hours. Pain is radiating to arm and jaw. Best
marker to confirm MI at this stage is
a. CPK
b. CPK MB = Ans (as no trop T mentioned)
c. LDH
51. A patient had heart-lung transplantation and was on immunosuppressive drugs now presents
with high grade fever, neck rigidity +ve kernig. CSF studies show an organism with clear
cytoplasm with a halo around it.
a. CRYPTOCOCCUS = Ans
b. EBV
c. staph aureus
d. klebsiella
e. CMV
52. In Shock which is the most important mechanism which conserves the organism as a whole
a. baroreceptor
b. sympathoadrenergic system = Ans
c. CNS ischemic response
Option B is best bcz it conserves perfusion to vital organs while CNS ischemic response comes at
end stage when perfusion to brain is low, it causes widespread vasoconstriction, which causes
organ failure.
53. Remnant of Umbilical Vein is
a. falciform ligament
b. ligament of teres = Ans
Teres is a part of falciform and is actually the remnant
54. Patient had hypovolemic shock, presents with signs of Acute Renal failure. Most likely damage
to which part of the nephron is greatest
a. PCT = Ans
b. descending
c. ascending
d. DCT
e. Collecting duct
55. A school going boy develops Right sided flank swelling and lumbar pain, US shows
Hydronephrosis of the right kidney while the other is normal. Which of the below is most likely
the cause of it
a. Urethral stricture
b. ureteral stricture
c. posterior ureteral valves = I marked as no other was found to be more appropriate
d. vesico ureteric reflux

56. A patient had an Accident with fracture of surgical neck of humerus. Most likely involved nerve
is
a. axillary = ANS
b. radial
all nerves options
57. Lymph node of inner breast drains into all except
a. inf phrenic nodes
b. supraclavicular nodes
c. internal thoracic nodes
d. pectoral nodes = ANS
58. Most common fracture of femur at age of 60 is
A. femur head fracture
b. intra capsular fracture
c. neck of femur fracture = Ans
d. shaft fracture
59. Epidural Space Ends at
a. S2 = Ans
60. Left gastric artery is a branch of
a. Celiac Artery = ans
61. Structure accompanying esophagus in diaphragm is
a. left vagus
b. right vagus/vagal trunk = Ans
c. phrenic nerves
d. azygous vein
62. Inhibitors of Prostaglandins are
A. Corticosteroids = Ans
b. aspirin
63. Hall mark of Adreno cortical insufficiency is
a. Hyponatremia = Ans
b. hypokalemia
c. metabolic alkalosis
d. hypocalcemia
64. Fast Pain travels by
A. A delta fibers = Ans
65. Sympathetic nerves ending at Inferior hypo gastric plexus have root value of
A. L1-L2 = Ans

66. Detrusor muscle of bladder supplied by


A. pelvic Splanchnic nerves (parasympathetic)
67. Pectoral group of axillary nodes drain
a. Upper half of trunk
b. Upper half of upper limb
c. Major part of the breast = Ans
d. Lower half of trunk
68. Regarding Vagus Nerve
a. Exits through passing middle part of jugular canal = Ans
b. Exits thorax by passing with Aorta
other options cant recall best may be A
69. Fragile X syndrome is
a. Trinucleotide repeat disorder = Ans
70. Most common cause of congenital inherited defects
a. Multifactorial = I marked
b. Autosomal dominant
71. A patient presents in OPD with vesicular eruptive rash which is starting from left temporal side
and ends near medial side of left eye. Rash is painful, diagnosis of Shingles is made. Which of the
following nerve may be involved
a. Ophthalmic nerve = Ans
b. Superior orbital nerve
c. Supra trochlear nerve
d. Maxillary
e. Mandibular nerve
72. Acrosome is formed by
a. Golgi bodies = Ans
b. Endoplasmic reticulum = I marked bcz I forgot it despite reading :D
73. External urethral sphincter related to prostatic:
a. Apex
b. Base = I marked
c. Posterior
d. anterior side
the question doesnt mentioned prostate or prostatic urethra, it was written Prostatic in Q.
74. Small particles are cleared by
A. Cilia in terminal bronchioles = Ans

75. Highest PCO2is in which vessel


a. Pulmonary artery = I marked
b. IVC = should be this bcz O2 consumption is highest in lower body and low in head.
c. SVC
76. Not isotonic to plasma is
a. 5% dextrose water
77. Lumbar triangle anteriorly bound by
a. Posterior border of External Oblique = Ans (SNELL)
78. Ulcer 2nd part of duodenum at posterior side artery eroded is
a. Gastro duodenal artery = ans
79. Slowest growing tumor of thyroid
A. Papillary carcinoma = ans
80. Commonest dislocation of TMJ is
A. Anterior = ans
81. A man has short trunk and longer arms, this disease is ________ .
a. Autosomal dominant disorder = Ans
82. Type 3 reaction is
a. Arthus reaction = ans
83. Myelin formation in CNS is from
a. Oligodendrocytes
I HAD 2 Questions of this in paper 2, straight forward Q & A
84. Mandibular branch of trigeminal nerve exits cranial cavity through
a. Forman ovale = ans
85. Heme in blood binds with
a. Haptoglobulin = ans
b. Hemopexin
86. Cerebral blood flow is regulated by
a. PCO2 in venous blood = Ans
b. pH of CSF
87. Eparterial bronchus is
a. Rt superior = ans

88. Epidural sinus in spinal canal


a. Longitudinal venous plexus = I marked other 2 options dont remember
b. CSF
c. Is between Dura and arachnoids space
89. Most common congenital anomaly of head is
a. Cleft lip & palate = ans
90. A woman head Radical mastectomy after recovery unable to raise arm above her head,
structure damage is
a. Long thoracic nerve = ans
91. Anterior relation of left kidney which is directly in contact with kidney without any membrane
a. Lesser sac
b. Pancreas = ans
c. Duodenum
d. Left colic flexure
92. Structure passing above lower head of Pancreas
a. SMA = ans
93. ICAM and VCAM are.
a. adhesion molecules = ans
94. Derivative of 2nd arch is
a. Stylohyoid = ans
rest were 1st arch derivatives
95. Adverse effect in long term use of glucocorticoids is
a. Osteoporosis = ans
rest were irrelevant
96. Branches of brachiocephalic trunk are.
a. Rt common carotid and Rt Subclavian
97. If LCX is occluded what will be the effect
a. Posterior of left ventricle be more affected = ans
b. SA nodal block will occur
c. Post interventricular septum 1/3 will be infarcted
98. SA node is pacemaker of heart because
A. It lies at the bottom of atrioventricular junction
b. It lies at the top of RT atrium = ans
c. It has a plateau
d. Its fibers have fastest conduction

99. Heinz bodies are present in


G6PD
100. 2 year old child was playing with toys when he suddenly become suffocated and cyanosed for
few seconds. Mother took him a hospital on CXR he had ate a small coin, where will it be lodged
most likely
A. Rt lower basal segment
101. A young child inhaled a peanut where it will be lodged most likely
a. Lower lobe of right lung
these were 2 diff questions with same answers both in paper 2
102.Intrinsic factor is secreted from
Gastric fundus
103.Virus causes
Alteration of protein synthesis
104.Moral support to patient in our society comes from
Family
105.A patient is binge drinker of Alcohol presents to ER with C/O RT side hypochondrial pain. US
shows fatty liver, Biopsy shows councilman bodies in hepatocytes.
Apoptosis
106.Feature of Atlas is
Absent Body
107.A patient is being evaluated for TB. The best investigation which will diagnose TB is
a. Caseous necrosis on Biopsy
b. Epitheloid Cells
c. Demonstration of Acid fast bacilli on ZN stain
108.Middle meningeal artery
Enters cranial Cavity through foramen Spinosum
109.What parameter is increased in Pregnant women
a. TIBC = ans
b. Serum ferritin
c. PCV
d. Hb
110.Nerve supply to pericardium is from
Ans = phrenic Nerve

111.Highest energy compound is


a. Fats
112.Pubic bones join anteriorly by
a. Secondary cartilaginous joints
113.Vein having no valves is
Ans= SVC
114.Following vein is a tributary of Portal vein
a. Superior rectal vein = ans
b. Hemiazygous vein
c. Inf epigastric vein
d. Left renal vein
e. Lumbar veins
115.A lady has difficulty in standing from a sitting position but can walk normally. Muscle involved is
Ans. Gluteus Maximus
116.A detached embolus from deep veins of leg will go first to
a. IVC
b. Rt Atrium
c. Lt ventricle
d. Pulmonary veins
e. Pulmonary artery = Ans
117.Defect of RBCs in Hereditary Spherocytosis
Ans. Cytoskeleton abnormality
118.Mixed venous blood sample can be obtained from
a. SVC
b. IVC
c. Rt atrium
d. Rt ventricle
e. Pulmonary arteries = ans
119.Long term glucose control can be best detected by
Ans. HbA1C
120.Glycocalyx is
a. Polysaccharide moiety on bacterial cell wall
b. Carbohydrate chain on cell wall = I marked this
rest options didnt remember

121.Basic drug binds with


ans. Alpha glycoprotein
122.Thyroid drains to
Ans = deep cervical nodes
123.Pregnant women with viral hepatitis. Best inv. to detect is
Ans. GGT
124.1/3rd of TBW is
Ans. ECF
125.Which hormone level will fall after drinking water
a. ADH = Ans
b. Angiotensin
126.Maximum blood pressure is at
a. Renal artery
127.Most common cause of K shift from ICF to ECF
Ans. Vigorous exercise
128.Glucose is main source for
ans. Neurons
129.Oxytocin and ADH originate in
a. Hypothalamus
130.Most common association of ADPKD is
a. Renal failure = ans
b. Cerebral hemorrhage
131.Muscle contracting due to sadness is
a. Platysma
132.Clostridium botulinum causes
a. Flaccid paralysis
133.Nerve accompanying superficial temporal artery is
Auriculo-Temporal Artery
134.Sarcoma differs from Carcinoma in
Increased Vascularity = ans

135.Patient on table heat loss mainly by


a. Conduction = ans (new ppl read this topic from ganong or Guyton)
b. Radiation
c. convection
136.Acidic urine in Tubules will cause
a. increased secretion of CLb. Decreased HCO3c. Dec K+secretion
d. Reduced H+ reabsorption ( I marked).
e. Aldosterone inc or dec dont remember
137.Baby born with congenital cataract, mother had most likely which intrauterine infection
A. Rubella
138.Left adrenal vein drains to
left renal vein = Ans
139.Lower Esophageal Veins drain to
A. Portal Vein
140.Left lung has groove for
a. Ascending aorta
b. Descending aorta
c. Arch of aorta = ans
d. Azygous vein
141.Deep vein thrombus 1st lodges in
a. Branches of pulmonary artery = ans (these were 2 Ques. In paper 1 slightly diff. But ans same.)
b. Rt atrium
c. Rt. Ventricle
d. IVC
142.Patient had TB, developed pleural effusion. Needle inserted will be along
upper border of lower rib = ans
143.Cause of pyogenic peritonitis is
a. E. Coli = ans
b. Bacteriodes
c. Klebsiella
d. Staph aureus
144.Metaplasia is
one cell type replaces other with functional change = ans

145.A patient had a coronary bypass developed A-fib now presents in ER with signs of stroke. Brains
shows which type of necrosis
a. Liquefactive = ans
146.HIV affects
CD-4 Cells T = ans
147.Labor working in hot weather collapses. Most likely abnormality found in him will be
a. Hyponatremia = ans
b. Hypervolemia
c. Hyperkalemia
d. Hypercalcemia
148.Carcinoma compared to adenoma has
a. Capsule
b. Invasion = ans
c. No loss of differentiation
d. Vascularity
149.TB resists immunity. In this case Antibodies are found
a. Attached to cell
b. Bound to organism
some other options were too, cant recall
150.Iron is stored in parenchyma under normal conditions is
Ferritin = Ans
151.Regarding saliva
a. Has inc Na as compared to plasma
b. Low K & HCO3 in comparison to plasma
c. Hypo tonicity is more than plasma = ans
152.Osmotic pressure depends upon
a. No of particles = ans
b. Diffusion potential
c. Charge on membrane on either side
153.Diffusion potential depends upon
a. Surface area of membrane = ans
b. Charge on membrane
c. Charge on either side
d. From low conc. to high conc.

154.From 3rd to 9th month of pregnancy, source of estrogen is


a. Corpus luteum
b. Granulosa cells
c. Placenta = ans
155.Strength provided to wound during healing is by
a. Collagen 1
b. Collagen 3 = ans
156.Granulosa cells are found in
a. Corpus luteum
b. Ground substance
c. Graffian follicles = I marked this or resembling one to this opt
d. Follicular epithelium
157.Esophageal constriction is by
a. Left atrium = ans
158.Regarding Rt Atrium
a. Forms lower border of sternocostal surface = ans
b. Is the posterior most structure
c. IVC ans SVC junction is visible at outer surface
d. Is supplied by LCX
159.Regarding bone formation
A. Newly laid bone is in the form of osteon = I marked other options not remember
160.Vibrations during Rapid Ventricular filling produces
A. 3rd heart sounds = ans
b. 4th heart sound
161.TPR is directly related to
a. Capacitance
b. HR
c. Vasomotor tone = ans
162.Exposure to hydro-carbons cause
a. Acute non lymphocytic leukemia
b. CML
c. AML
d. Liver cancer
163.Cause of mesothelioma is
Asbestos = ans

164.Sub acute bacterial endocarditis is caused by


Ans. Strep viridians
165.Cornayae diphtheria mode of transmission is
a. Oro-fecal
b. Respiratory droplets = ans
c. Skin contact
d. From spores
166.Inhibin is produced by
a. Sertoli cells =ans
167.Lymphatic nodules absent in
A. Thymus = ans
B. Spleen
c. Payers patches
168.Rapidly adopting fibers are.
a. Pacinian = ans
b. Meissners
c. Merkel
d. Ruffini
e. Free nerve endings
169.Regarding vagus nerve in GIT
a. Stimulation causes relaxation of ano-rectal sphincter
b. Contraction of longitudinal muscle group for peristalsis = ans
c. Decreased acid secretion
d. Constriction of oddi sphincter
170.In case of Anterior duodenal perforation, fluid will go to
a. Left paracolic gutter
b. Rt sub hepatic space
c. Rt subphrenic space
d. Rt Para colic gutter = ans (SNELL REF)
171.In case of posterior perforation of 1st part of duodenum, fluid will be collected into
a. Lesser sac
b. Rt subphrenic space = I marked
c. Rt Sub hepatic space
d. Rt paracolic gutter

172.Antibodies are formed by


Ans. Plasma cells
173.Glucocorticoids cause
a. Increased production of protein
b. Hepatic gluconeogenesis = ans
c. Dec amino acid metabolism
174.Which hormone acts through phospholipase C
a. Hormone causing increased blood flow in penis
b. Causing water reabsorption in kidneys Adh V2
c. Constriction of vasculature by ADH V1 = ans
d. ANP
e. Angiotensin causing vasoconstriction
175.Sensory supply to skin over lower half of deltoid is supplied by
Ans. Axillary nerve
176.Preganglionic autonomic fibers are
Ans. B fibers
177.Cells which are enlarged in adenoma of parathyroid
a. Chief cells = ans
b. Oxyphil cells
c. Both
178.Genital branch of genito-femoral nerve
a. Passes through deep inguinal ring = may be
b. Gives branches to anterior scrotal wall = I marked B = cant recall other options
179.Characteristic of IgM
Ans. Is a large molecule (pentamer)
180.Highest triglycerides % is in
Ans. Chylomicrons
181.Efficiency of a drug is
a. Efficacy = ans
b. Potency
182.Tension in muscle is sensed by
Ans. Golgi tendon

183.A patient presents to you in emergency. His pH is 6.95 HCO3 4, glucose 500mg/dl. You
administer ______ insulin
Ans. Regular insulin (couldnt remember was this question in paper or not)
184.Isthmus of uterine tubes is
A. Below the fundus
b. Is the narrowest part of tube = ans (THIS ONE TOO I DONT REMEMBER it was in paper or not)
185.Genioglossus supplied by
Ans. C1 fibers through hypoglossal nerve
186.Chondorcytes change into osteocytes
A. Thyroxin
b. Somatotrophs = ans
187.Patient with lower backache and pain radiating to posterior thigh. The most likely cause is
A. Disc herniation at L4-L5
b. Disc herniation at L5-S1 = ans.
188.Maximum Ventricle is filled during
a. Rapid Inflow
b. Isovolumetric contraction
c. Atria systole = Ans.
Reason is 80% Ventricle is filled during rapid inflow, but 20% comes from atrial kick which
becomes like this 80%+20% = 100% filled
189.Pulse Pressure
A. Depends upon Rt Ventricular stroke Volume
B. Low in high Cardiac output state
C. Increase in peripheral arteries than systemic arteries
D. Depends upon compliance of Venous system = I marked
190.Embryonic period is
Ans. = up to 8th Week (this ques. Was in paper but with slight wording change)
191.Vertebrae is derived from
Ans, = Sclerotome
192.Common site of fracture of Rib is
Ans. Angle

193.A soldier had gunshot wounds, presents in ER with massive hemorrhage, ideal fluid to be
transfused is = CPSP Demo Ques.
A. Whole blood for 3 days = Ans.
B. PCV
C. Crystalloids
D. Colloids.
194.At pelvic brim, ureters are anterior to
A. Common Iliac = ans
B. Ext. Iliac
C. Gonadal artery
D. IVC
195.A female lady underwent surgery for Uterine tumor. Next day she felt abdominal pain on LLQ.
Structure most likely injured is
A. Ureter = Ans
B. Bladder
C. Ovaries
D. Kidney
E. Sigmoid colon
196.Cremasteric Fascia derived from
Ans. Internal Oblique muscle
Couldnt recall other Questions, 183/184 were or were not in the paper but rests of the questions were
in 1st shift Radiology 5th Sep.
After compiling this I pray we all may pass. Amen
Remember in my prayers. If you want to add or correct any question, please message/inbox me, itll be
appreciated.
Regards: DR. MUHAMMAD QASIM NAEEM

Gynae and Obs 7th September 2016 (evening)


Compiled by : Dr Tara Ahmed (Just Remember in prayers)
Thankyou Round Ligament Ayesha Noor & Stubborn Veena
This was the most pathethic and tragic exam in the history of cpsp!
1.Lateral attachment of urogenital diaphragm;
a. Sarum n coccyx
b. Obturator internus membrane
c. Inferior ramus of the pubis and ischial ramus
d.ischial tuberosities
e. tip of coccyx

2. The most appropriate therapy for endometriosis is


a. Dnazol
b. Norgesterol
c.Oral androgens
d.Oral contraceptive
e.Raloxifine
3. important buffer of blood?
a. Hb
b.hco3
c.Acetate
d.protien
e.phosphate

4.Content of fat in male is?


a. 10%

b.15%
c.40%
d.60%
e. 17%
5. Erythropoetin is secreted by?
a. endothelial cells of the peritubular capillarie
b. jg cells
c. adrenal cortex
d. glomerulus
e. loop of henle

6. 1.The richest source of vitamin ...?


a. whole milk
b. butter
c. margarine
d.liver cord oil

7. concentration of which of the following hormone doesnt increase during pregnancy:


a) PTH
b) cortisol
c) GH
d) prolactin

8. Size of ureter is
a. 20-25 cm
b.25-30 cm
c.30-35cm

d.35-40 cm
e. 15-20cm

9. Blood supply in midline of flap of forehead ??


a.frontal branch of frontal artery
b. paired supra trochlear artery
c. optahlmic branch of trigeminal artery
d.facial artery
e. paired Supra orbital artery

10. A fetus was aborted .it was sent to forensic lab .how can the viability be checked .tht for how long
the fetus was viable.?
a.Fetus weight 200g
b.Eyes laterally
c.Fetus CRL >250 mm
d.Absence of wrinkles
e.Absent toe nails

11.what is absent in C1?


a.Spinous process
b.Arch
c.transverse process
d.Lateral mass
12. difference between roots of right lung from left lung?
a.No of primary bronchi
b.No of arteries
c.No of veins
d.No of nerves plexus

13. cisplatin has the following side effect


a.Nephrotoxicity
b.Neurotoxicity
c.hepatotoxicity
d.nausea , vomiting

14.How does body adapt to cold temperature in long term.


a .Internal pyrogens
b .Chemical thermogenesis
c. Formation of brown fat fat
d. peripheral vasodilation
e.pheripheral vasoconstriction

15.which of the following factor deficiency cause coagulation?


a.5
b.7
c.8
d.10
e.12

16. hyperbaric oxygen treatment is most useful in?


a.CO poisoning
b.Anemia
c.Polycythemia
d.decompression sickness
e.Gas gangrene

17. 2 feet away from ileocolic junction at antimesentric border


a.urachal fistula
b .meckel diverticulum

18. tunica albuginea


a.Contain testes only
b.Derivative of parietal peritoneum
c.derivative of visceral peritoneum
d, B.covers ductus deferens

19. Costco diaphragmatic recess


a.seen only on left side
b.Wide open space in centre
c.
d.

20. deep ring of inguinal canal attaches


a.Fascia tranvesalis
b.Tranversua abdominus
c.External oblique
d.internal oblique

21.An anesthetist wants to give his patient most potent muscle relaxant who is about to undergo
cesearation so that she may relax completly.he would give
a.Bupivacaine
b.Lidocaine
22. Root value of femoral nerve
a.L2 ,L3, L4
b.L2,L3,L4,L5,S1,S2
c.L1 ,L2

23. A patient complains of arthritis and discoid rashes. Which of the following would be specific?
a. Ds Dna

b. Ana
c. Ama
d.Anti ccp

24. At the end of marathon race , a person has ???


a. high insulin low glucagon
b . high glucagon n low insulin
c. high cortisol
d. high insulin and glucagon
e.low insulin and glucagon

25. A 25 years old married female came to you with history of amnoerhea for 3 weeks.
her beta hcg shows that her pergnanacy test is negative. What could be the cause?
a. ectopic pregnancy
b.anovulation
c. primary hypothalamic disorder
\
d.Anorexia Nervosa
26.test for difference of two means = T test
27. In Tuberculosis, antibodies are
a. cell bound
b. inside cell
c. absent
28. Toxoplasmosis is
a. parasite

b helminth
c.bacteria
d. virus
e.Fungi

29. Virus acts on cells by:


a;Damaging cell membrane,
b;Damaging nuclear membrane,
c:Invloving protien synthesis,
d;Changing protien structures.
e;Changing protien metabolism
30. Aspirin and other NSAIDS work by acting on
a.Lipoxygenase
b.Cyclo-oxygenase A2
c.HMG Co A reuctase inhibitor
31. Which of the following is a spore forming bacteria?
a. clostridium
b.cornybeacterium
c. Chlamydia

32. microscopic feature of malignancy


a.Invasion
b.Pleomorphism
c.N/C ratio

33. A patient with pulmonary embolism and extensive tissue injury. What is the most probable
cause of his dx.
a. Dvt

b. Thromboembolism
c. Atril myxoma

34. Aldosterone acts on?


a. Collecting duct
b. Dct
c. Cortical collecting tubule

35. Plasma proteins return to normal after


a.2 -3 days
b..7-8 days
c.10-12 days
d.12-14 days

36. An abnormally tall boy of 14 year age consults his physician for his abnormal height and
secretions from his mammary gland. X ray skull shows enlarged pituitary gland. Histological
examination of pituitary is kukely to show:
a. Decreased number of pituicytes
b. Hessing bodies seen in karge number.
c. Increased no of basophill cells.
d. Increased no of chrimophobes
e. Increased no of eosinophills

37. prolactin raised with infertility what will be cause


a.microprolactinoma
b.macroprolactinoma
c.pituitary tumor

38. Diagnostic test for typhoid in 2nd week is :


a.Blood culture and widal test .
b. blood culture
c. stool culture
d.widal test
e. urine culture

39. Substance which act as both hormone and nurotransmittor?


a. adrenaline
b.cortisole
c . acetylcholine
d. thyroid
e. Aldosterone

34.Stimulation of parasympathetic nerves to the heart decreases heart rate by


a. Increase in permeability of K thru ventricles
b. Incraese in permeability of k through sa node
c. prolonging of AV nodal delay

35. Starling's law of the heart:


a.Does not operate during exercise
b.Is not obeyed by the failing heart
c.Explains the increase in heart rate during exercise
d.Explains the increase in cardiac output when venous return increases
e.Explains the increase in cardiac output due to sympathetic stimulation

36.A lady presented with per vaginal bleeding,history of bleeding disorder and having
a dead fetus ..first step of management
A.blood Transfusion
B. Evacuation of uterus
C.platelet transfusion
d.iv heparin

37. Platelets
a.half life of 10 days
b. Increase after spleenectomy
c.granules contain serotonin
c. Stimulated by growth hormones

38. The ascent of kidney is prevented by


A.external iliac artery
B.internal iliac artery
C.superior mesenteric art
D.ivc
E.inferior mesentric artery

39. which of the following structure has both intra n extra pelvic extension
A.broad ligament
B.ovarian ligament
C.transverse cervical ligament
D.round lig of ovary
E.round ligament of uterus

40. most potent chemotactic factor


A.c3a
B.c5a
C viruses
D.arachidonic acid metabolites

41. Rbcs are


A.biconvex
B.nonfragile
C.largest of all cells
D.have glycolytic enzyme activity

42. lysosome contains


A.glucose
B.Na+
C.ca +
D.acid phosphatase
E. Water

43. gram -ve infection causes septic shock in which of the following tracts
A.biliary tract
B.colon
C.small bowel
D.urinary tract
E.genital tract

44. flexor reflex by


A.Muscle spindle
B.Monosynaptic
C.messner corpuscle
D.noxious stimulus
E. By Tapping with hammer
45. study on sexually transmitted disease,one group of boys had penile ulcers and the other
group had no ulcers,non ulcerating leisons were d/t?
A.treponema paliidum
B.chlamydia
C.HPV
D.ducreyi

E.Gonorhea
46. .barr body test in saliva is diagnostic for
A.klinfelter syndrome
B.gonadal dysgenesis
C.turner syndrome

47. You are testing efficacy of unknown unnamed drug A and drug B in an infection...then
randomnly allocated to pts recieving both of them..it is an example of
A.double blind/placebo
B.single blind
C.triple blind.
D.cross sectional study

48. anemia of maturation failure..immature cells in periphery


A.iron def anemia
B.normocytic
C.pernicious anemia
D.microcytic anemia
49. the difference of osmolarity b/w interstitial fluid and plasma is
A.1mosm/L
B.1.5Mosm/L
C.3 Mosm/L
D.5 Mosm/L
E.no difference

50. .local cause of delayed wound healing


A.infection
B.ischemia
C.vit c deficiency
D.cortisol

51. tricuspid auscultation area


A.2nd rght intercostal space
B.4th left intercostal space

C.mitral area
D.lower half of right border of sternum
52. aortic hiatus aneurysm compresses
A.thoracic duct and vagus nerve
B.aygous vein
C.phrenic nerve
53. pulmonary wedge pressure
A.measure of pulm artery pressure
B.measure of rght atrial pressure
C.measure of left atrial pressure

54. about csf


A.It is an ultra filtrate of plasma
B.provides nutrition to cns
C.formed by arachnoid granulation
D.separated from interstitial fluid by blood brain barrier

55. growth hormone stimulation


A.rem sleep
B.wakeful condition
C.hypoglycemia
D.free fatty acids

56. which one of the following is mesenchymal in origin


A.choristoma
B.hemartoma
C.sarcoma
D.carcinoma
E. Choriocarcinoma

57. in the first step of thyroid hormone synthesis iodine attach with
A.albumin
B.thyroxin
C.thyroglobulin

D.tyrosine
E.hb

58. tumor with all three germ layers


A.teratoma
B.adenocarcinoma
C.hamartoma
D.sarcoma
E.seminoma
59. female child,breast fed developed jaundice on 2nd day hb 9 bilirubin 8 unconjugated
A.mthr is RHPostive
B.breast fed jaundice
C.physiological jaundice
60. .bifurcation of common carotid artery is felt at
A.anterior border of sternocleidomastoid at the superior border of thyroid cartilage
B. post border of sternocleidomastoid in posterior traingle
C.at level of trapezius
D. post border of sternocleidomastoid at the level of hyoid

61. an adult in sitting position.resonance can be auscultated from back of the chest till
A.6th rib
B.8th rib
C.10th rib
D.11th rib
E.12th rib

62. rbcs
a.lack mitochondra
b.half life of 180days
c.released from bone marrow as mature erythrocyte
d. Present on mhc 2
e.are the largest cells of the body

63. .scenario of increase serum calcium,irregular bone,also bone pain...which of the following
causing it
A.PTH
B.GH
C.vit D deficiency
64. .lysosomal storage disease with mental retardation and blindness
A.taysachs disease
B.granulomatous disease

65. measured by spirometry


A.residual volume
B.TOtal lung capacity
C.residual volume
D.vital capacity

66. the obstetric nerve block cause urinary incontinence,anaesthetized muscle is located in
A.pelvic diaphragm
B.urogenital diaphragm
C.superficial perineal pouch
D.none of the above

67. .true about superior parathyroid gland


A.is at mid of posterior border of thyroid at first tracheal ring
B.its position changing
C.is not enclosed in pretracheal fascia
D.all of above

68. which structure has 3 layers of muscels


A.fundus of stomach
B.esophagus
C.one of the above
D.all of the above

69. which of the following is an example of primary active transport


A.H+K pump
B.Na glucose co transport in intestine
C.Na-Amino acid co transport in intestines
D.none of the above
70. alpha receptors effects
A.contraction of radial muscle of iris
B.vasodilation in skeletal muscle
C.increase in activity of gut
D.decreased heart rate

71. regarding parasympathetic supply of heart


A.Left superficial cardiac plexus
B.vagus nerve
C.sympathetic system
D.Left deep cardiac plexus
72. which of the following supresses growth hormone release
A.Puberty
B.sleep
C.somatomedin
D.starvation
E.stress

73. A female with regular cycle of 21 to 23


Days will most likely be ovulated on which of the following days
A.12-14
B.14-18
C.7-9
D.10-12

74. trypsinogen is activated by


A.enterokinase
B.peptidase

C.amylase
D.acid phosphatise
E. HCL
75. most early sign of vit A deficiency is
A.keratomalacia
B.cheilosis
C.hyperkeratosis
D.diminshed dark adaptation

76. a female presents with jaundice her lab reports show Alt 17,alk phosp.75 and retics
10.cause of jaundice
A.hemolytic anemia
B.pernicious anemia
C.iron def anemia
D.all of the above

77. true regarding cavernous sinus


A.occulomotor trochlear and opthalmic of trigeminal present in lateral wall
B.connected posteriorly to facial vein
C.drain directly into straight sinus
D.has abducent nerve in medial wall
E.Related Laterally to sphenoid Sinus

78. A man with bee sting,develop swelling and acute infalmaation.mediator for it?
a.Histamine
b.Bradykinin
c.prostaglandin
d.leukotrine
e. Esinophills

79. severe blood reaction occur


a.A+ To 0+
b.O+ To B+
c.A- To A+
80. Pregnant lady with mitral stenosis, pt.developed dysponea.orthopnea and difficulty to
breath. Most probable cause would be?
a.Right heart failure
b.Pulmonary htn

81.on a study about diabetes between 2 groups it was found


Women with mean 20050
Men with mean 20510, this means
a. Women with low mean and high standard deviation
b.Man with high mean and low standard deviation
c.Both have no link
d. Case control
e. Cross Sectional study

82. which is obsevatinal study


a.Case control
b.Cohort
c.cross sectional study
83. breast outer quadrant lymph drainage
a.Ant.pectoral
b.central
c, apical
d. posterior

84. testicular lymph node drainage


a.Paraaortic
b. superficial inguinal
c. deep inguinal
d. para umbilical
85. superficial perineal poch content
a.Bulbourethral gland
b.Perineal nerve
c,Sup.transverse perineal muscle
d. internal pedundal vessels

86. branchial cleft


a.Open laterally
b.Open laterally ant.to sternocleido mastoid

87. draining sinus and granuloma is the feauture of


a. aspergillus
b. nocardia

88. Which of the following helps in fotmation of the bones?


a. osteoblast
b.osteoclast
c.osteoprogenitor cells
d. fibroblast

89. more central value of no. In series


a.Mean
b.Median
c.Mode
90. A child since he was born was having problem of urine leaking from ventral surface it may
be due to?
a. Epispadiasis
b. Failure of urogenial folds to close

91. Lacunar ligamnt forming medial boundary , fibres enrolled to form.


a.Inguinal ligament
b.Transverse ligament
c.Relected ligament
92. Malignacy is caused by which of the following
a.Silicosis
b. Asbestosis
c.Smoking

93. ADH is inhibited by


A.Alchol
B.increase Body water

94. Sympathetic autonomic system true is....


a.Adrenergic preganglionic
b. Thoracolumbar outflow ....
c.Short postgang fibres ....
d.pre and post ganglionic short fibres
95. Most common bacteria involved in std is

a.hpv
bChylmydia
c.Gonnorhea
d.Hsv

96. Person wid bloating abdominal pain very foul smelling stools .First Investgation would be
a.Cumplete stool Dr
b.Stool cs
c.Lipid profile
97.most Comon mediator of fever
a.1L-1
b.Tnf.
c.. IL 6

98. increase in ECF osmolarity Lead to


a. stimulate plasma vol and osmoreceptors nd stimulate adh
b. Inc adh and also nhibit osmoreceptors nd vol
c.inhibits adh and stimulate vol and osmoreceptors
d.stimulate adh and inhibit vol and osmoreceptors

99. Person having history of trauma on the left upper arm he develop swelling in arm that was painful
for 3months it was well circumscribed 4 cm and on xray appeared bright then it gradually deceased in
one year most Likely ....
a.Metaplasia ...
b. Hyperplasia ....
c,.Neoplasia .....
d.Dysplasia ...
e.Hypertrophy

100. Irregular growth of bone is due to the effect of which of the following?
a.leydig cells ....
b.Ant pituary hormone effct ....
c.Pth

------------------------------------- Paper 1 1 --------------------------------------------------(The second phase of this horrible examber for which we are still traumatized and the scars seems like
never healing! . The poor victims of 7th September evening shift)

1. Drug via oral route having aqueos absorption ....


a.Digoxin ...
b.ephidrine .. ..
c.Phenobarbital
d.diazepam
e.carbamizapine
2. Cerebellum ....
a.Iniates movemnt ....
b.stabalizes movement and coordination
3. Most common cause of bronchogenic carcinoma
a. Tobacco
b. Asbestos
4. Female pale lethargic .didnot take k any antenatal care .... During delivery she bleeded
heavily .... Which nutrient deficiency would be present in both baby nd mother ?
a .Vit k
b.Iron
c.calcium
5.

permeation is defined as
a.tumor invasion to blood vessels
b.tumor invasion to lymphatics
c.tumor spread to adjacent tissues
d.distant metastasis

6. Pt presented wid leg ulcer and severe pain nd on blood examination howell jolly bodies present
wats the diagnosis ?
a.Sickle cell ...
b.Hereditory spherocytosis
c.beta thalasemia
d.alpha thala

7. Longst acting insulin ?


a.Porcine lente
b.Human lente ....
c.bovine Ultra lente
d.bovine semi lente . (one more something like nph)
8.

7yrs child anemic for two years with blood investigations shows decreased hb 6.5 also Hbf 97%
... Hba2 4% no hstry of bld transfusion . spleen palpable.
a.Thalasemia major
b.thalasemia trait
c.herditory sphero
d. hereditary persistence of Hbf

9.

Female wid Warty lesion on vulva ..


a.condyloma ...
b.Papilloma ....
c.Herpes

10.

long tubular structure having mucosa and transitional epithelium


a.ureter
b.kidney
c.urethra

11. During sexual act secretion from vagin are effect of


a. Bombesin
b.Relaxin
c. Progsterone
d.gnrh
e. Parasympathetic stimulatiom
12. Measurement of amount of lead in body
a.absorbed from git
b.absorbed from respiratory tract lower lobes.
c.absorbed from skin
d.absorbed from nasal passages.
13. About zygote implantation
a.28cell stage.

b.3-4days after fertilization


c.invasion of endometrium by syncitiotrophoblast
14. trigone of bladder
a.ureters entering as oblique slits
b.devoid of peritoneum.
15.

Carbonic anhydrase act on which part of kidney


a.pct.
b.early dct
c.cortical collecting ducts
d.]calyces.

16.

person given nacl one litre infusion wat will happen


a.osmolality increases in plasma
b.osmolality decreases in urine
c.na increases in plasma
d.na increases in urine

17.

isonizid not metabolised in people with deficiency of


a.glucoronyl tranferase
b.hyloroinidase
c.pseudocholinesterase
d.xanthine oxidase
e. amine oxidase

18.

cells having oxident stress as oxygen radicals were produced which one of the following can
decrease this effect
a.nadph oxidase
b.glutathione oxidase

19.

in end stage renal failure wats affected on prolong basis


a.erythropoitin decreased
b.thyroid hormone decreased

20.

propofol contraindicated in
a.hypersensitivity
b.renal disease
c.cardiac disease

21. Toxic effect of lignocaine?


a. Prolong seizures for 9-7 hours
b.hypotension]

c.Perioral anesthesia
d.rash
22. data from smaller study on one group of people is combined with data from larger study what
type of data is this
a.case control
b.randomised control trial
c.cohort
d.meta analysis

23. cerbral metabolic rate of oxygen


a.15-20ml/min/kg
b.2-3 ml/min/kg
c.3-4ml/min/kg
24. hyaline disease of new born
a.maternal diabetes
b.maternal eclampsia
c.prematurity
25. urea is formed by liver in the body to get rid of
a.ornithine
b.arginine
c.ammonia
26. fracture of clavicle inner side n medial half , upward displacement takes place it is due to which
muscle
a.sternocleido
b.subclavicular
c.trapizius
d.perctoralis major
e.deltoid

27. inner medial lower quadrant of breast doesnt drain into


a.anterior axillary
b.inf phrenic
c.supraclavicular

28. which nerve in urogenital triangle is damaged


a.post cutaneous nerve of thigh
b.ilioinguinal
c.genitofemoral
d.femoral
29. protamine side effect
a.hypotension
b.bleeding tendency
30. after abdominal hysterectomy on 3rd day post op there is serosanguinous amount of fluid from the
wound wats the cause
a.wound infection
b.wound dehiscence
c.wound hematoma

31. 7yrs old child with coughing and mucus inc eisoniophills plug on xray it shows hyperinflation and
infiltrates whats the diagnosis
a.cystic fibrosis
b.bronchial asthma
c.gastric aspiration
d. hypersensitivity pnumonitis
32. normal menstrual blood flow is less than
a.80ml
b.120ml
c.40ml
d.60ml
33. women during menstruation uses tampons presents .she develops vomiting diarrhoea
hypotension high grade fever .which organism is involved in it?
a.staph aureus
b.e colli
c.gardnerela vaginalis
d.chlaymidia
e.gonnorhea

34. remnant of epiphiseal plate


a.cortical bone
b.trabecular bone
c. spongy bone (this is a poor recall)
35. locally malignant cancer
a.melanoma
b.adrenoblastoma
c.pleomorphic adenoma of parotid
d. Adenoid cystic Carcinoma
36. blood pumped by heart per min
a.750ml/min
b.6lit
c.5lit

37. esmolol is given in anesthesia because


a.its selective beta blocker effect
b.decreses pain
C. doenot increase icp
d. doesnot cause rebound hypertension
38.

bladder ca
a.schistosomia hematobium
b.japonicum
c.mansoni

39. Cotisol decreases.......glucose utilization in tissues


40. before Ovulation maturation of follicles occur by action of
a.Fsh
b.Estradiol
c.Gnrh
d.Lh
41. Superior and inferior mesenteric arteries anastomose on
a. marginal
b. colic

c. left ileocolic
d. sigmoidal arteries

42. after amoebic infection . rectal biopsy was done now to confirm which test would be done
a. pas
b. electron microscope
c. perl reaction
d. antibodies against amoeba

43. lethal hepatitis in pregnancy. Hep E


44. Thickish white curdy discharge. Candidasis
45. Erythroblastosis. ........ Type 11 Reaction
46. Read Null hypothesis :/
47. trachea ?
a. related ant to thyroid.
b.fibrocartilagenous ring
c. Ends by dividint at the lower border of T4

48. Blood supply to rectum is from.... superior and imferior rectal arteries
49. Captopril... mechanism of action? Inhibits angiotensin 11
50. Tetanus
a.exotoxinz has lethal effect
b.exotoxin afect on cerebral
c.no role of active immunity
d.trismus is rare.

51 Which one is precancerous condition


a..myelodisplastic syndrome
b.cervical erosion
c.osteoporosis
d.psoriasis
52. Parasympathetic effect on lungs
a.inc compliance
b.inc restrictive work
c.inc anatmicl dead space
d.,inc elastic work

53. cell responisble for innate immunity r activated most commonly by


A carbohydrare seq in bacterial cell wall
B cytopasmic protein of bacteria
C eosinophil
D glucocorticoid
E pollen

54. regarding cilia:


a)stereo cilia are a type of cilia
b)can be seen on light microscopy
c)can be seen on electron micscopy
d)has absorptive function
e. cannot be seen under light microscopy
55. tb is closely associated with .....hypersensitivity
56. Alpha adrenargic antagonist...
a. doxazocin
b. clonidine
57. intermediate mesoderm give rise to

....a. Somites
b.Muscle
c.Heart tube
d. genitourinary system
58. Paramyxovirus; in control group causes for which aso titre was being done
a.Glomerulopnephritis
b.Sydenham chorea
c.Arthritis
d.Carditis
e.erythema marginatum
59. New anti hypertensive drug want to check potency..
Method use were... just remember one option sequential trial. Kindly confirm
60. Stroke volume increases?
a. Venous dilation
b.Inc in heart rate at constant cardiac out put
c. venous compliance
d.inc in blood volume
61. a 7 years old boy had pharyngitis with halitosis and peritonsillar abcess
a. cornybacterium diphtheria
b. streptococcus
c.staphylococcus
62. Spinal cord with dura arachnoid and pia mater in adults terminates at the level of lowr
border of....... L1?
63. coccygeal ligament at the level of?
a.L1
b.L2
c .S1
d.coccyx

64. u did a study and it rejected null hypothesis it mean


A)There z difference
B) there z no difference.
C) study of research is not valid.
D. ur research is not powerful enough

65. Abducent passes arise through


A.midbrain
B.pons
C.medulla

66. Which structure connects skeletal muscle to bone ?


a.Tendon
b.ligament
c.joints
67. true about spinal nerve ?
a. derived entirely from nueral crest
b. pas tru intervertebral formina
c. dorsal root contain both sensory n motor supplies

68. annulus pulposis forms.-anulus fibrosis (confirm please)


69.

Guys! Ive tried my level best to recall all the questions I could. Some mistakes maybe there,
correction will be highly appreciated. For boosting up your confidence SEPTEMBER 6,2016 Evening
Paper I was 90% repeat and paper II was around 65-70$ repeat.
Due to poor recall could only recall the options that were most suitable; pardon me for that

1. Regarding Aphasia; Lesion in: Temporal lobe, there was on option regarding
brocas or wernickes
2. Fascia enclosing masseter muscle
Deep Cervical Fascia
3. There was a scenario or TURNER; female with webbed neck and short stature
4. A Scenario of KLINEFELTER; Male with XXY
5. Decrease in core body temperature results in: Vasodilation (I marked) or Shivering
6. Virus cause disease by;
Alteration in protein synthesis
7. Regarding node of Ranvier:
no Myelin
8. Highest TGs concentration:
Chylomicrons
9. Regarding hemorrhagic infarct:
Lungs (I marked) ; Liver was also in options
10. Type of collagen present DURING wound healing:
Type III
11. Regarding reticular connective tissue; Spleen*
12. Smoking working in tyre factory, cause of transitional cell carcinoma: Smoking >
Aromatic
13. Supernumerary ribs:
compression of Brachial plexus and subclavian
14. Diseased vs non diseased:
Case control
15. Gardener with linear something:
Cutaneous larva migrans
16. Repeat question of a diabetic who died despite aggressive measure:
Mucor
17. Insulin inhibited by:
B blockers
18. Vitamin in fatty acid oxidation
biotin
19. Aggressive sympathetic drive:
Bronchodilation
20. Most important response in shock as a whole?
CNS ischemic or
Sympathoadrenergic
21. Increased BT is observed in:
vWF deficiency
22. Regarding Renal vein:
Anterior to Aorta
23. Left adrenal vein drainage:
Left Renal Vein
24. Retromandibular vein:
Maxillary with superficial temporal
25. Chromatids align at equatorial plain:
Metaphase
26. Child presented with proteinuria of >3gm, cause on edema?
Decrease colloid
osmotic pressure
27. Natural Defense Against Cancer:
Apoptosis

28.
29.
30.
31.
32.
33.
34.
35.
36.
37.
38.
39.
40.
41.
42.
43.
44.
45.
46.
47.
48.
49.
50.
51.
52.
53.
54.
55.
56.
57.
58.
59.
60.
61.
62.
63.
64.
65.
66.

Step in Apoptosis:
Activation of caspases
Child with macrocytic type CBC:
Intrinsic factor antibody>B12 level
Elastic cartilage present in:
Epiglottis
Regarding Hyaline cartilage:
Articular surfaces
Mother having measles, child at risk of:
Cataract
Mother delivered a baby with cataract, cause? Measles
Sound produced in Ventricular filling; S3
Which of the following decreases serum osmolality? ADH or 20% albumin or hypertonic
dextrose
Thiazide diuretic;
Hypokalemia
Fracture of Lumbar vertebra:
Conus medularis syndrome
True about osmosis?
Passive process or its osmolality depends on
particles number
Regarding nucleus?
EuchromAtin is transcriptionally active
In severe dehydration
decrease in total body water
A patient with severe dehydration drinks 2L tap water:
Inc ICF (I did) or inc ECF
Global cause on infective blindness:
Chlamydia > bacteria (probably)
Internal carotid artery:
anterior choroidal branch to. dont
remember or passes beside cavernous sinus (I did)
Leukocyte Adhesion
LFA1
True about Leukocytes
Phagocytic in blood or move in and out of
vessel
Repeat case of EXTRAHEPATIC CHOLESTASIS
Preganglionic autonomic Fibers
B type
Regarding Vit A?
T cell Receptor bind with
MHC
Secondary ossification center present in?
Associated with intestinal malignancy H. Pylori
Decrease ESR
Increase Albumin
Patient aspired on lying down
Right lower lobe
Irreversible step of glucose metabolism?
G6P to f6P?
In second degree heart block
Atrial rate greater than ventricular (maybe)
41% hematocrit
41% RBC WBC Platelets
Structure embedded in cerebral cortex?
Insula?
SA node location
upper part of sulcus terminalis
Ureteric bud development
Mesonephric duct
Patient presented with chest pain of 2-4 hours duration, all baseline was normal next
investigation
CPK-MB
Mechanoreceptor of Warm
Long receptive fields
Gracilis and cuneatus damage
Astereognosia
PICA supply
dorsolateral medulla
Cells having most telomerase activity Germ Cells
Most drugs are metabolized in
Liver
Pseudomembrane colitis
C. difficile

67. Reason of tuberculin test negative


taking immunosuppressant
68. Case of PARKISON with all associated symptoms like tremor, bradykinesia, rigidity gait
problem, lesion in
Substantia Nigra
69. Heart Rate of 120
RR interval 0.5sec
70. 60 years old farmer with Cushing symptoms
Adrenal Adenoma
71. Malignant tumor
Metastasis
72. 2*2 table
Chi Square or T test
73. Which augments the effect of phagocytes?
Complement or Opsonin
74. Metastasis occurs because
loss of E cadherin
75. Allele
Non identical genes
76. Hypovolemia results in
Dec carotid sinus nerve
77. Typhoid investigation in first week
Blood Culture
78. Thiamine deficiency causes
Peripheral neuropathy
79. Increase in GFR and RPF
afferent arteriolar dilatation
80. Cimetidine effect
Inhibit hepatic enzymes
81. Inducers of liver enzymes
Phenobarbital
82. 50 years post-menopausal lady
Inc. FSH and LH
83. Severe transfusion reaction occurs
A+ to O+
84. Which drug increases LES tone and gastric motility
Metoclopramide
85. NK Cells
Kills virally infected cells without sensitization
86. Cushing features with Raise ACTH
Cushing disease
87. Mediators of Pain
Bradykinin and PG
88. Young patient dyspnea on lying down Retrosternal goiter
89. Breaking bad news
Information in crispy and acceptable way
90. QRS complex occur
Prior to ventricular systole
91. Alpha and gamma motor neuron
UMN supply
92. Atheroma development major factor Presence of cholesteatoma? Alcohol? HTN?
DM?
93. Response to secure bleeding due to dermal abrasion
vasoconstriction
94. Increase thyroid hormone cause
increased in gluconeogenesis
95. There was a female with malar rash
Anti DSDNA
96. Regarding somites
Arise from both sides of notochord?
97. Early sign of Vit A deficient
Night blindness
98. Partial mole
69XXY
99. Lady presented with uterine bleed due to coagulation failure secondary to miscarriage
Evacuation platelets and FFPs
100.
Regarding sensory fibers
May pass or may not pass reticular depending
upon modality?
101.
Motor Fibers
Supply alpha and beta simultaneously
102.
Young female with decrease hb, Dec mch, hbA 94% hbA2 was 4%
Thalassemia trait
103.
A smoker patient, was having interstitial lung disease and pleural plaques
Asbestosis
104.
Compression of sciatic nerve level was asked

105.
106.
107.
108.
109.
110.
111.
112.

Best measure of GFR estimation


Creatinine or Inulin
A patient with BP200/120 and glucose >200, what to give
FEV/FVC
0.8
Bronchial asthma
decrease FEV1
th
4 heart sound correlates with Atrial depolarization
Rapid acting hypoglycemic drug
Lente or NPH
Autonomic supply to mid gut T8-t12* or T6-t9
Contraction in GIT muscle is due to

Lisinopril

a. Spontaneous Na channels
Spontaneous Ca Channels
Hormone and slow waves
Neuronal and Slow waves
Neuronal and action potential*
113.
100cm of ileum resected
decrease absorption of bile salts
114.
How to monitor a patient on warfarin PT, APtt, INr
115.
Increase Bt and Aptt normal PT
vWF defect
116.
Ganglion found at c7 vertebra
Cervicothoracic
117.
Patient having constricted pupil, unable to dilate it
cervical sympathetic
rd
118.
Regarding esophagus
upper 1/3 striated
119.
Color Blindness due to
Cones defect
120.
In severely dehydrated child, venesection was performed; most likely nerve to
be damages?
Saphenous
121.
Osmotic diuretics act upon
PCt
122.
A known case of HCV, histology of liver shows eosinophilic cytoplasm, nuclear
clumping
apoptotic hepatocyte
123.
A patient presented with fatigue, mcv104, alt 74 ast 80 Hepatitis C> B12
124.
A patient presented with irregularly irregular pulse, p wave was replaced by
irregular waves
Atrial Fibrillation
125.
Digoxin given in
Atrial Fibrillation
126.
Atonic bladder
Damage to parasympathetic below
sacral
127.
Most radiosensitive tumor
Lymph node
128.
A patient presented with acute exacerbation of COPD, Po2 45, Pco2 51, pH 7.31,
Bicarb 26
Type I failure or Type II?
129.
A patient doing exercise, increased respiration. Which of the following will
activate, that remains inactivated at normal respiration
Ventral resp group?
130.
MAO of Aspirin
inhibit COX
131.
Ondansetron control vomiting by
antagonism pf 5HT3
132.
Itching is carried by
Type C
133.
Viral infection and purpura
antibodies to Platelets (ITp)
134.
Abduction and medial rotation or arm Teres Major
135.
Microscopic feature of malignancy
Invasion> pleomorphism
136.
Brain perfusion at rest
50ml/100gm

137.
138.
139.

Brain auto regulatory pressure?


50-150mm Hg
Coccygeal ligament
L1
On right lung bronchoscopy first section to be visualized?
Anterior branch of superior or superior bronchus?
140.
Narrowest point of esophagus
Cricopharynx
141.
Point mutation occurs in
Sickle cell disease
142.
A patient diagnosed as amyloidosis having plasma cell disease, amyloidosis is
due to deposition of
Amyloid light chain
143.
Which of the following function through cAMP Glucagon
144.
Which of the following passes thru aortic hiatus? Azygous vein
145.
Norepi secreted at
post ganglionic sympathetic
146.
Patient from Baluchistan presented with fever, hepatosplenomegaly, and has
history of fly bite
Kala Azar
147.
Intracranial veins connected to extracranial Emissary veins
148.
A child with congenitally missing dorsalis pedis artery, blood supply to dorsum
of foot maintained through
peroneal artery
149.
A child got blow at elbow, elbow joint displaced, now he is having loss of
sensation in little finger
ulnar collateral ligament
150.
35 years old male patient died suddenly, autopsy finding nil, tox screen shows
cocaine
contraction band necrosis
151.
Another question was on CONTRACTION BAND NECROSIS
152.
Edrophoneum most common side effect
Nausea?
153.
In females, neck of bladder lies
above urogenital diaphragm
154.
Perineal innervation
S2,3,4
155.
Death due to C. Botulinum
Respiratory Muscles Paralysis
156.
A female having blurred vision, decreased muscle activity, other neurologic
nerve involvements, cant recall
demyelination of nerves
157.
Antibodies to post synaptic Ach receptors Myasthenia Gravis
158.
Loss of sensation of lower teeth and chin Inferior Alveolar Nerve
159.
Alpha 2 agonist used in ICU for sedation Dexmedetomidine
160.
Decrease release of Oxygen from Hb
decrease in Temp
161.
Hyperuricemia is side effect of ATT
Pyrazinamide
162.
AIHA diagnosed by
Anti Globulin Antibody Test
163.
One question was on OCCULOMOTOR Loop
164.
C. Diphtheriae
secretes Exotoxin
165.
Benign tumor is
Warthin
166.
On biopsy, endocervix show squamous cells Metaplasia
167.
Which structure loops around arch or aorta Left RLN
168.
Sound heard in left 3rd ICS
Aortic A2
169.
Regarding kidney transplant
HLa1 more important than HLA2
170.
Scenario was given 5 years arthritis history with congestive cardiac failure
features cardiomegaly and glucose was 167 what lab finding in this patient you will find
ESR 79 or raised Ferritin
*Ferritin was in thousands
171.
Benzodiazepine cause
Dependence

172.
173.

Highest pH is seen in
Pancreatic Secretion
A child presents with perianal pruritus; which cells will be elevated
Eosinophils
174.
Gastric HCL is necessary for
Converting pepsinogen to pepsin
175.
In pleural fluid aspiration in midaxillary line which structure will not be pierced?
Levator Costrum
176.
Left Ventricular Failure cause
Aortic Valve pathologies
177.
A patient having a lot of pizza and burgers
Hypertrophy of Steatocytes
178.
There was one question on coronary circulation
anastomose at
arteriolar level or run in respective grooves
179.
Nerve root of sciatic nerve
180.
Which of the following cause paraneoplastic syndrome Small Cell CA
181.
A patient was presented with Malar Rash and other symptoms of SLe what
other finding can be found?
Pericarditis
182.
Blood flow to muscle during exercise
Inc. local metabolite

183.

Exophthalmos occurs due to

TSI

Remember me in your prayers (Y)


Surgery paper 2016 5th Sep 2016 EVENING
1.

Lymphatic from the lateral quadrant of the breast mainly drain into the
A. Inferior deep cervical lymph node
B. Inter pectoral lymph node
C. Pectoral (anterior) lymph node
D. Retrosternal lymph node
E.

2.

A student wanted to study relation ship between increase in temperature and duration of surgery, to
evaluate results which statistical test he should apply?
A.
B.
C.
D.
E.

3.

Supra clavicular lymph node

Student t test
Anova test
Mc witneys
Chi squared
Regression Analysis

Action of Gluteus Medius & Minimus


A. Abduct the hip joint & Rotate the thigh medially
B. Adduct the hip joint and rotate the thigh laterally
C. extend the hip joint
D. flex the hip joint

E. helps in sitting in squatting position


4. Heavy smoker complains of dyspnea chest pain cough hemoptysis since 3 months. Bronchoscopy
reveals fungating mass obstructing medium sized bronchus. Which type of tumor?
A. squamous cell carcinoma
B. small cell carcinoma
C. large cell carcinoma
D. Bronchiolar carcinoma
5. Skeletal Muscles are attached to Bones by:
a. Anchoring filaments
b. Fascia
c. Ligaments
d. Tendons
6. Sugar in Urine will be detected if its concentration in blood becomes
a. 250mg/dl
b. 200mg/dl
c. 180 mg/dl
d. 150 mg/dl
e. 375mg/dl
7. Dorsal Nucleus of Vagus Nerve lies at
a. Pons Lower Lower Part
b. Medulla
c. Pons Upper part
d. Midbrain
8. 2% solution of 4ml lidocaine will contain How much Lidocaine
a. 80 mg
b. 160 mg
c. 8 gm
d. 40 mg
9. 0.85% Nacl Solution contains how much NACL
a. 85mg/100ml
b. 85 mg / 1000 ml
c. 850 mg/100ml
d. 8.5 mg/100 ml
10. A person with Chest pain having ST Elevation in Lead V4 only. which artery involved
a. LAD
b. RCA

c. Anterior Interventricular
d. LCX
e. Diagonal
11. Which of the following is Ectoparasite
a. Lice
b. Fleas
c. Ticks
d. Bed Bugs
12. Before final composition of Concentrated Urine, which part of Tubular fluid contains least osmolar
solution
a. PCT
b. Collecting Duct
c. Ascending loop of Henle
d. Descending loop of henle.
e. Distal Convulated Tubule.
13. Best Term to describe to describe a condition when a cell gets extra number of chromosome other than
haploid
a. Non disjunction
b. Trisomy
c. Euploid
d. Aneuploid
e.Polyploidy
14. Alveolar Surface is kept dry because of
a. Tight Junctions
b. Surfactant
c. Negative Interstitial Pressure
d. Macrophages
15. Histoplasmosis affect which system mainly
a. Respiratory
b. Reticuloendothelial
c. Cardiac System
d. Nervous System
16. A female delivered a baby and she was started on warfarin as she developed dvt. The next day she has a
purplish patch on her right thigh what can be the cause
A. lupus anticoagulant
b. warfarin overdose
c.protein c def
d.proteins def
17. Grade 3 Tumor, % of Undifferentiated cells is :

a. 25 %
b. 50-75%
c. >75%
18. In a Pre-OP patient after inducing General Anesthesia, heat will not be produced due to:
a. Muscle tone is lost
b. Na/K Pump Blocked
19. Initial symptoms of Bupivacaine Overdose
a. Arrythmia
b. Unconsciousness
c. ringing of ears
20. Cytoskeleton communicates with Extracellular environment through
a. Intermediate Filament
b. Integrins
c. Cadherins

21. True about Glycocalyx:


a. Carrier Protien
b. Carbohydrate Moiety
c. Lipid Membrane
22. Acoustic Neuroma is a tumor of
a. Cochlear part of CN 8
b. Vestibular part of CN 8
c. Facial Nerve
d. Trigeminal nerve
23. In young age, Limb growth will be affected if fracture passes through
a. Epiphysial Plate
b. Epiphysial Line
d. Diaphysis
d. Metaphysis
24. Cause of Death after Liver transplantation due to which pathogen
a. CMV
b. HBV
c. HDV
d. HCV
e. HEV

25. In VIT K deficiency, which factor will be depleted first and foremost
a. Factor 7
b. Factor IX
c. Factor 10
d. Protein C
e. Protein S
26. SKIN GRAFT will be most successful in which of the following conditions
a. Excessive Burns
b. Agammaglobenemia
c. Terminal cancer patient
d. Liver failure
27. Which cancer invades large blood vessels
a. Clear cell ca of Lungs
b. Clear cell ca of Kidney
c. Bronchogenic Ca
d. Transitional cell Ca
28. In Old what happens
A. Dec Vital Capacity
B. Inc Cardiac Output
29. Neutral absorption of sodium occurs at which part
a. PCT
b. Thick ascending limb of loop of henle
c. Thick descending limb
d. Cortical collecting duct
e. DCT
30. Breast Atrophy in elderly females, Major cause
a. Estrogen & Progestrone deficiency
b. estrogen deficiency
31. Major cause of fatty liver in our society
a. Hep B & Hep C
b. alcohol
c. increase fat intake

32. Unable to evert and dorsiflex, which structure injured


a. Common peroneal nerve
b. deep peroneal nerve
33. Scortal Swelling after injury, contents of swelling extends upto abdomen but not thigh, Most common
cause of this swelling in scrotum
a. Injury to Bulbar Urethra
b. injury to penile urethra
c. membranous urethra
d. neck of urethra
34. Mediolateral Episiotomy, which part at risk of being getting injured
a. External Anal sphincter
b. Bulbospongiosis
c. levator ani muscle
d. perineal body
35. Axillary Artery
A it lies post to pectoralis minor muscle
B lateral chord lies posterior to it
C starts at the lateral border of pectoralis minor
D all the chords are lateral to it
36. Ecchymosis and bleeding hx in fisherman
a. Vit C Deficiency
b.Vit b12 def
c. Folic acid def
d. Iron def
37. Chromosome 9:22 abnormality
CML
38. Function of Vitamin A
Forms Rhodopsin
39. Lymph drainage in Cervical Ca
External & Internal Illiac lymph nodes
40. Dec in ESR due to
Inc Albumin
41. Recurrent Laryngeal nerve injured on both sides, Which muscle still intact

a.thyrohyoid
b.postcrici thyroid
c.cricothyroid
d.cricoarytenoid
42. Gastric Lymphoma due to
H Pylori
43. The features of BCC
a. metastasis
b. benign tumor
c. borders ar eundermined
d. Locally malignant
44. Posterior wall of Rectus sheath b/w Symphisis pubis & Abd
Arcuate line
45. Bundle of His Blood supply
a. RCA
b. LCA
c. LAD
d. Circumflex
46. Head of Femur Blood supply
Medial & Lateral Circumflex Arteries
47. Major Cause of Epiphyseal Tumor
a. Osteoblastoma
b. Osteosarcoma
c. Osteoid Osteoma
d. Ewing Sarcoma
48. DIC is initiated by
a. Tissue Thromboplastin
b. fibrinogen
49. Seventy years old male develops sudden loss of power in both right upper n lower limb with angle of
mouth deviated to the left during talking. The most likely site of lesion is in
a.Internal capsule
b.pons
c.medullaoblongata
d.midbrain
50. Sciatic nerve injured which structure will still flex the knee

a. Short head of biceps femoris


b. long head of bicep femoris
c. popliteus
51. Patient died of Stroke, which type of necrosis
a. Liquificative necrosis
b. Caseous necrosis
c. coagulative necrosis
52. Median Umbilical Ligament is remnant of
a. Urachus
b. umbilical artery
c. umbilical vein
d. allantois
53. Which structure will be damaged in Lumbar Puncture
a. Ligamentum Flavum
b. Posterior longitudinal ligament
c. Anterior longitudinal ligament
d. Pia matter
54. Scenario with Vit K deficiency, Vitamin K not absorbed, which factor decreased
a. Prothrombin
b. Protein C
c. Protein S
d. Factor VIII
55. Low estrogen pills will cause
a. Breast Ca
b. Lungs Ca
c. Endometrial Ca
d. Deep venous thrombosis
e. Hepatic Adenoma
56. Pregnant lady with raised SGPT, history of visit to Remote village, which hepatitis
a. Hep A
b. Hep E
c. Hep B
d .Hep C
57. Which Ca will not metastasize
Basal Cell Ca

58. Isthmus of trachea lies in front of


A. 2nd 3rd and 4th Tracheal Ring
B. Rings 3 4
C. Rings 2 3
59. 50 years old hypertensive patient presented with severe chest pain radiating to back
suddenly collapsed On autopsy what can be seen
a. Medial Necrosis of Aorta
b. schemic necrosis
c..athero sclerosis
60. Abdominal Aorta
a. Gives Renal Arteries at L2
b. bifurcates at L5
c. bifurcates at T5
d. lies at the right side of IVC
61. Loss of Supination & Flexion after Stab Injury to Inner Arm
a. Musculocutaneous
b. ulnar
c. radial artery
d. axillary artery
62. Regarding Ciliary body
A. Pigment epithelium produces aqueous humor.
b. Pigment epithelium of retina is continuous With pigment epithelium of ciliary body
C. Non pigment epithelium produces aq. Humor
D. Supplied by short ciliary arteries
63. Minerals are least found in
a. Pulses
b. Cereals
c. Tubers
d. Green Vegetables.
64. Patient had previously Episodes of Dizziness and Diplopia, patient presented now in Acute Comatosed
state, Cause :
a. Carotid Artery Thrombosis
b. Basilar Artery Thrombosis
c. Arachnoid Hemorrhage

65. Which of the following lymphoid organ receives lymph via SUBCAPSULAR SINUS
a. Thymus
b. spleen
c. Lymph node

66. True regarding rectum is :


a. lies in front of S1.
b. is 9 inches long
c. is biconvex wall in front of s2,3,4
d. also supplied by median sacral Artery
67. Infection anterior to pretracheal fascia will spread to:
a. Anterior Mediastinum
b. Superior medianstinum
c. Middle Midiastinum
68. Regarding abdominal angina which artery is involved
a..inferior mesentericartery
b..aorta
c..superior mesenteric artery
69. premature baby born with
a..PDA
b.patent foramen ovale
c.right atrial pressure increases
70. Which harmone causes fatty acid synthesis and protein synthesis
a.cortsole
b.thyroid
c.glucagon
d..insulin
71.Most common salivary gland tumor
a.salivary
b.submandibular
c.parotid
72. Regarding anal canal
a.supplied by both superior n inferior rectal arteries

b.inf rectal artery part of portalsystem


c.external anal sphincter is involuntary
d.ha stwo lateral curves
e.sup inguinal lymph nodes drain the upper half of anal canal
73. Patient started on anti coagulant how to moniter further
A. Aptt
B. Clottingtime
C. Pt
74. Trachea commences at
A. C4
B.C5
C. C6
D. C1
75. A female of reproductive age presented with DVT. She gave history
Of taking some kind of pill what is the cause of her condition
a. Thrombo embolism
b. Analgesics
c..OCPs
76. 45 years old man presented with chest pain not related to respiration
Myocardium
77. Tongue is deviated towards right side , lesion is
a.
b.

Right hypoglossal nerve


Left hypoglossal nerve

78. Deep part of the parotid fascia forms


a. bucco pharyngeal membrane
b.stylomandibular ligament
c.sheno mandibular ligament
d.stylo hyoid ligament
79. first Lumbrical
a.bipennate
b.attached to proximal phalynx
c.origin near the radial side off first meta carpal bone
80. In a terminal resection of ileum 100 cms of ileum was resected. What will be the out come?
a.less absorption of bile salts
b.less absorption of bile acids
c.more absorption of bile salts
d.increase sodium loss

81. Regarding inguinal canal


A. Absent in infants.
B. Extends from anterior superior iliac spine to pubic tubercle.
C. Roof formed by conjoined tendon
82. Greatest clearance is of
a. PAH
b. insulin
c. Glucose
d. Creatinine
83. Regarding cardiac plexus
a.formed by both symathetic n parasympathetic nerves
b.thoracic splanchnic nerves are the main nerves
c.starts at trachea bifurcation
d.present at the base of heart
e.anterior cardiac nerves are the main nerves

84. Baroreceptors
a.hypotention
b.respond to rapidly increasing arteriolar pressure
c.respond to rapidly decreasing arteriolar pressure
85. Insulin is inhibited by
a. secretin
b. glycogen
c. beta agonists
d. beta blockers
86. Biaoavailibity of minerals is lowest in
a.roots
b.green leaves
c cereals
d pulses
e.Tubers
87. Drug drug interaction by
Both pharmacokinetics and pharmacodynamics
88. Regarding sweat glands
Cuboidal epithelium
89. Bioavailibility is not affected by

a.age
b.liverdisease
c sex/gender
90. Dimercaprol
a.increases clotting
b.decreases clotting
c.decreases bleeding time
D. Increase bleeding time
91. For sodium neutralization
a. k
b. organicions
c. Cl
d. mg
e. hco3
92. Amino acid containing sulphur
Cystein
93. Sodium regulation is through
A osmoreceptors
b.sodium absorption
c.Adh
d.Reninangiotensin
e.Aldosteron
94. Circumflex artery supplies
a.ant surface of heart
b.left atrium n ventricle
c.rightatrium
d.leftatrium
e.leftventricle
95. 41%haematocrit contains
a.rbcs
b.rbcs,wbcs,platelets
c.platelets.....
96. Venous blood as compare to arterial blood has
a.low ph
b.more packed cell volume
c.increasepo2
d.increase ph.
97. During stab injury

Ipsilateral lung collapse n ipsilateral chest expands


98. A person got lacerated injury on his elbow what will be the initial affect of blood
haemostasis
vasoconstriction
99. during a one km running the venous return to the heart is by
a.valves in the leg veins
b.pumps in the legs
c.skeletal muscles contraction
100. right atrial pressure is increased this will lead to
a.inc cardiac output
bheartrateinc
c.inccontractionofrightventricle
101. Regarding occupational diseases increase risk of problems with exposure to
a.coalmines
b.asbestos
c.silicons
d.textile industry
102. Eversion of foot leads to ligamentum fractured
a.lateral ligament
b.planatarisruptured
c.Deltoid ligament
103. A farmer while spraying suddenly got collapsed with salivation from his mouth n
broncho spasm. What is the specific anti dote
a.atropine
b.neostigmine
cphysostigmine
d.Pralidoxime
104. A person was walking bare feet suddenly stepped upon some sharp object. He quickly
removed his leg due to which reflex
a.flexor response
b.mono synaptic reflex
c.multi synaptic reflex
105. Opsonization due to interaction of
FC portion of igG and C3b
106. Morphine is used in

Terminal cancer illness


107. Inferior wall MI
Right marginal

108. Most appropriate about Middle meningeal Artery


a.pterion is the surface marking
b.its branches form grooves on the under surface of skull
c. Involved most commonly in Extradural Hematoma
d.enters skull thru foramen spinosum
109. vagotomy done for
a.decrease motility of gut
b.decrease gastric acidity
c.decrease pepsin
d. decrease gastric acidity n pepsin motility of gut
110. pain of Gastric ulcer is due
Greater splanchnic nerve involvement
111. medial wall of ischiorectal fossa is formed by
a. Levator ani muscles
b. external anal sphincter
112. Double membrane bounded Organelle
a. Nucleus
b. RER
C. Mitochondria
d. ribosomes
113. Most radio sensitive tumor is
a. seminoma
b. glialglioma
c. craniopharyngioma
114. Man is intermediate host in
A. Hydatid cyst
B. Beef tapeworm infestation
c. trichomoniasis
d. filiriasis
e. hook worm
115. a young man fell from two storey building and now he has urinary incontinence. lesion is at

s2s3s4
116. S2 differs from S1 by
a..s2 has increase duration
b.s2 has decrease pitch
c s2 has inc frequency
d s1 has increase pitch
e s1 has dec duration
117. a child presented to opd with 104 fever for the last 4 days. He has been passing cola
colored urine for the last one day. He has bee n on anti malarials too. His lfts are deranged with increase
unconjugated bilirubin. What is the most probable diagnosis
a.drug induced jaundice
b.black water fever
c.paroxysmal nocturnal haemaglobinuria
dhaemolytic uremic syndrome
e G6pd

118. 46XX associated most probably with which of the following conditions
a. Adrenogenital Syndrome
b. Mixed Gonadal dysgenisis.
c. Congenital Adrenal Hyperplasia
119. Turnor syndrome with
a.AR
b.AD
c shows secondary infertility
d stunted growth is the main feature
120. ECF is increased by infusion of
A hypotonic saline
B normasaline
C hypertonic saline
d dextrose water
121. Peri capsular sinuses in
a.lymph node
b thymus
c palatine tonsils
122. Weight bearing line passes through
a. Ischial tuberosities
b. pubic crest

c. ischial spines
123. A 14 years old girl presented with right iliac fossa pain. pain associated with vomiting and
was radiated to per iumbilical region. Which segment is involved/or pain radiation due to
a. ilioinguinal nerve
b.iliohypogastric nerve
c.t12level
d L2level
e T10 level
124. microscopic slide of liver shows portal vein hepatic artery and bile duct at periphery
and central vein in the centre. What is this called
a.hepatic lobule
b portal acinus
c hepatic acinus (also called liver acinus)
d Simple acinus
125. EEG of a patient shows
A increase frequency of waves in active mind than in deep sleep
B is symmetrical in both hemispheres
C small amplitude of waves in intense thinking
D determine intelligence
126. what is produced in muscles
A phospholipids
B creatinine
C urea
D uricacid
E glycogen
127. About sertoli cells
A produce testosterons
B rapidly deviding cells
C release androgen inhibiting factor
D break blood testis barrier
E reinkes crystals
128. In CRF what is least likely to be presented?
A low ph
b hypo phosphatemia
c enlargedparathyroids
d metabolicacidosis
ehypocalcemia
129. On autopsy of CRF patient which organ was hypertrophied
A. Para thyroid glands

b. adrenal glands
c. thyroid gands
d. pituitary gland
130. Strongest layer of small bowel
A submucosa
B musoca
C longitudinal layer
D circular layer
E lamina propria
131. chronic hepatitis histology on microscope shows
A fibrosis
b neutrophills
c lymphocytes
dlymphokines
132. sarcoma as compare to carcinoma
a. spread to bones early
b. in the form of clusters of cells
c. more sensitive to radiotherapy
d. Clumps/Clusters
133. Systemic antifungal is
Amphoterecin
134. Heparin inhibits
A organization of clot
B retraction of cot
C dissolution of clot
D propagation of clot
135. During pregnancy in which period teratogen affects fetus mostly
A 1 to 2weeks
B 3 to 8 weeks
C 22 to 36weeks
D last trimester
E 8 to 12weeks
136. muscle is prevented from tearing under action of which receptor
a.muscle spindles
b GTO
c ruffinis
d meissners
e phosphate bonds
137. maximum sodium entry in nerve fibre takes place at

A during depolarization
B during repolarization
C at nodes of Ranvier
D as the impulse propagates along length of nerve fibre
138. The valve which has more propensity toward diseases is
a. Mitral Valve
b. tricuspid valve
c. pulmonary valve
139. Non bacterial endocarditis due to
A rheumatic fever
B SLE
C streptococcal glumerulonephritis
D thrombo embolism
E neoplastic diseases

140. 23 years old boy came with mild jaundice. His hbsAg is negative, anti Hbc igM is
Neg, anti Hbe positive.Whats the current situation
a Acute resolving phase
b acute hepatitis
c window period
d chronic hepatitis
141. Difference between CSF and cerebral capillary is
a. increase ph
b carbohydrates
c increase pco2
d less proteins
142. Benign tumor among following is
a. Leiomyoma
143. Hypotensive pt admitted in ward wd h/o dysuria and high grade fever. His blood culture is positive
for pseudomonas. The disease is due to release of
A. endotoxin
B. exotoxin
Cproteases
D. Toxic shock syndrome toxin.
144. Highest diffusing capacity across respiratory membrane and in body fluids is
A helium
b carbon monooxide

c carbon dioxide
d nitrogen
e oxygen
145. Tumor suppresser gene
a. p53
b. ras
c. c-myc
146. upper motor neuron lesion leads to
A decrease tendon reflexes
B fasiculations
C inc muscle tone
D flaccidparalysis
147. which nerve passes through cavernous sinus
A. abducent nerve
b. trigeminal nerve
c. occulomotor nerve
148. premalignant growth
A sebborhic keratosis
B compound nevus
C intradermal nevus
D dysplasia
e junctional nevus
149. 35 years old girl has been using analgesic for backache. She presented with platelets in normal
range.wbc 7000 neutropenia. Her hb was 9 with small immature cells in peripheral smear. Diagnosis
a acute leukemia
b ITP
c aplastic anemia
d drug induced
150. Gastric emptying increases by
a.secretin
b.Gastrin
c.Cck
d.ach inhibition
151. local edema cause
Allergy
152. Exogenous antigens acted by interacting with
MHC class II
153. A lady presented with yellow eyes and she has got hepato megaly. Also xanthomas

present. What can be the cause in her


A AMA
B ANA
C sclerosing cholangitis
D lipidemia
154. Regarding PseudoHYPERparathyroidism?
A. because of Hypocalcemia
B. Because of PTHrP by various tumor
C. associated with Nephritic syndrome
D. because of Vit D def
155. Right ovarian artery drains into
IVC
156. Regarding biceps brachi
a. insertion on humerus
b. Arises from supraglenoid tubercle
c. pronation
d. extention of elbow
157. A 22 years old man came to Opd with ALT raised viral marker s were negative. He has got some problem
in the eye and Immunological tests were derranged. Whats the most useful investigation for him
A liverbiopsy
B HbsAg
cHCVvirology
d serum copperlevel
e cerulo plasmin level
158. a diabetic patient presented with increasing degree of progressive illness involving respiratory
system. Gastrointestinal. What can be the cause
a histoplasma
b acidbasebalance
c Mucormycosis
d hyperkalemia
e bloodglucose
159. A person got an accident and there was h severe bleeding from his body. He got
multiple transfusios but Ecg showed Changes after transfusion due to
A hypocalcemia
B .hyperkalemia
C hypokalemia
D hypermagnesemia
E PHchanges
160. Doctor patient relationship
Active listening

161. after adrenelectomy taste preference increases for


A sweets
b.Nacl
c sour
162. 60% of ventricular filling is because of
A Rapid inflow
B slow inflow
C Atrial systole
D slowejectionphase
E isovolumetricrelaxation
163. about follicular cancer of thyroid
a.vascular invasion
b psammoma bodies
c lymphaticspread
d hyperchromatism
164. protein on Rbc membrane for chloride exchange
a.spectrin
b. band 3
c.glycocalyx
165. Regarding cholangio carcinoma
Chlonorcis sinensis
166. primary syphilis initial diagnosis
a. mouthulcer
b.genital sore
c.tabesdorsalis
167. Dead space does not decrease in
a.shallow breathing
b.deep breathing
c.lung diseases
d.gravity
e.tracheostomy
168. chemical mediator of inflammation
a.histamine
b.serotonine
c.lymphokines
169. Adverse blood transfusion reaction occurs in
A o+ to A+

B o neg to A+
C A neg to A+
d. AB+ to A+
e Ab neg to A+
170. Regarding un coupling of Oxygen thats produced by
a.thyroid
b.norepinephrine
c.epiephrine
171. If a 70kg healthy adult loses one litre of blood within 5 minutes there would be
a.dec TPR
bi.ncGFR
c.inc venous tone
d.dec diastolic pressure
e.splanchnic vasodilatation

172. injury to pulmonary alveoli causing constriction of alveoli by


a.hypoxia
b.atherosclerosis
c.adenosine
d.prolonged oxygen therapy
173. Primitive streak mesoderm cells migrate Round cloacal membrane toform ant abdominal
wall cells. If this migration does not happen around cloacal membrane will rupture and open cranially as
a extrophy of bladder
b imperforate anus
c ectopic anal canal opening
d hypospadias
174. Diptheria toxin will affect most lethally on which organ
a. Larynx
b. Heart
c. Kidney
175. Sparing of the lateral part of the thenar eminence with conspicuous wasting of other small muscles of the
hand suggests a lesion of:
A. Ulnar
B. Median
c. C5 and C6 spinal cord segments

D. C8 T1 spinal cord segments


e. Radial nerve
176. blockage of beta adrenergics will result in
a. Decrease heart rate
b. heart contractility
c. insulin pdn from islet of pancreas
d. lipogenesis
177. Due to stress of surgery what will increase and cause vaso constriction
A. ACTH
. B. Serotonin
. C. Cytokines.
D. increase level of circulating catecholamines

178. Intraarticular disc of tmj involved.. Muscle paralyzed.


A. Lateral pterygoid
B. Med pterygoid
. C.Temporalis.
179. Femoral arterial pulse.
A. Mid inguinal point
B. Mid inguinal ligament.
C. Adductor canal.
180. Regarding posterior Communicating artery
A. Connects PCA to ICA above occulomotor nerve
B.Connect Pca below occulomotor nerve
C. Connect ica to basilar artery.
181. Child has episodes of bleeding nd echymosis since birth hb 9.8 mg/dl nd platelets
count was normal 180 x 10^3 bt prolonged
A. Hemophilia
B. itp
C. platelet functional defect
182. Corona radiata forms from
A. Germ cells.
B.theca interna.
C. Theca externa
d. Granulosa

183. All preganglionics neurotransmitter


A. Acetylcholine
184. Ppd false negative
A. Malnutrition
B. Immunosuppressant
185. Most important cell of chronic inflammation
A. Lymphocytes
B. Macrophages
c. neutrophils
186. patient with lung abscess develops meningitis, causative organism.?
A. Staphylococcus
B. Streptococcus

187. 2x2 table


A. T test.
B. Anova .
C. Chi square

188. How can doctor provide better management to patients:


a. By better communication
189. Which form of thyroxin is an active form
A. Reverse t3
B. Unbound form
C. Bound to albumin
D. Bound to prealbumin
190. Parasympathetic effect on Heart
A. Increase PR interval
191. which of the following augment the effect of immune system?
A. Complement System
B.opsonins
192. For chemotaxis

A. Lymphokines
B. serotonin
193. Testosterone main function
a. Grows hair on scalp
B. Inc muscle and bone growth
194. Sweat gland by.
a.sympathetic
195. Primary active transport
A. Enzymes
b. Carriers
c. Proteins
d. Pumps
196. Question regarding edema--A. Lymphatic blockage
197. Primary Brain vesicle is
Mesen cephalon
198. Test for typhoid fever in first week
a. Blood culture

You might also like